Top Banner
INSIGHTSIAS SIMPLYFYING IAS EXAM PREPARATION INSTA Tests 21 to 24 (GS) www.insightsonindia.com prelims.insightsonindia.com | mains.insightsonindia.com Telegram: insightsIAStips | FB: insightsonindia | TW: vinaygb | YT: INSIGHTS IAS BENGALURU | DELHI | HYDERABAD INSTA 75 Days REVISION PLAN UPSC Prelims 2020 Copyright © by Insights IAS All rights are reserved. No part of this document may be reproduced, stored in a retrieval system or transmitted in any form or by any means, electronic, mechanical, photocopying, recording or otherwise, without prior permission of Insights IAS. KEY & EXPLANATIONS
96

SIMPLYFYING IAS EXAM PREPARATION - INSIGHTSIAS · 2020-04-13 · Insta 75 Days Revision Plan for UPSC Civil Services Prelims – 2020 This document is the compilation of 100 questions

Jul 18, 2020

Download

Documents

dariahiddleston
Welcome message from author
This document is posted to help you gain knowledge. Please leave a comment to let me know what you think about it! Share it to your friends and learn new things together.
Transcript
Page 1: SIMPLYFYING IAS EXAM PREPARATION - INSIGHTSIAS · 2020-04-13 · Insta 75 Days Revision Plan for UPSC Civil Services Prelims – 2020 This document is the compilation of 100 questions

INSIGHTSIAS SIMPLYFYING IAS EXAM PREPARATION

INSTA Tests

21 to 24 (GS)

www.insightsonindia.com

prelims.insightsonindia.com | mains.insightsonindia.com

Telegram: insightsIAStips | FB: insightsonindia | TW: vinaygb | YT: INSIGHTS IAS

BENGALURU | DELHI | HYDERABAD

INSTA 75 Days REVISION PLAN UPSC Prelims 2020

Copyright © by Insights IAS All rights are reserved. No part of this document may be reproduced, stored in a retrieval system or transmitted in any form or by any means, electronic, mechanical, photocopying, recording or otherwise, without prior permission of Insights IAS.

KEY & EXPLANATIONS

Page 2: SIMPLYFYING IAS EXAM PREPARATION - INSIGHTSIAS · 2020-04-13 · Insta 75 Days Revision Plan for UPSC Civil Services Prelims – 2020 This document is the compilation of 100 questions

Insta 75 Days Revision Plan for UPSC Civil Services

Prelims – 2020

This document is the compilation of 100 questions that are part of InsightsIAS

famous INSTA REVISION initiative for UPSC civil services Preliminary examination

– 2020 (which has become most anticipated annual affair by lakhs of IAS aspirants

across the country). These questions are carefully framed so as to give aspirants tough

challenge to test their knowledge and at the same time improve skills such as

intelligent guessing, elimination, reasoning, deduction etc – which are much needed

to sail through tough Civil Services Preliminary Examination conducted by UPSC.

These questions are based on this INSTA Revision Plan which is posted on our

website (www.insightsonindia.com). Every year thousands of candidates follow our

revision timetable – which is made for SERIOUS aspirants who would like to intensively

revise everything that’s important before the exam.

Those who would like to take up more tests for even better preparation, can

enroll to Insights IAS Prelims Mock Test Series – 2020

(https://prelims.insightsonindia.com). Every year toppers solve our tests and sail

through UPSC civil services exam. Your support through purchase of our tests will help

us provide FREE content on our website seamlessly.

Wish you all the best!

Team InsightsIAS

Page 3: SIMPLYFYING IAS EXAM PREPARATION - INSIGHTSIAS · 2020-04-13 · Insta 75 Days Revision Plan for UPSC Civil Services Prelims – 2020 This document is the compilation of 100 questions

INSTA 75 Days REVISION PLAN for Prelims 2020 - InstaTests

www.insightsonindia.com 1 Insights IAS

DAY – 21 (InstaTest-21)

1. Bureau of Energy Efficiency star labeling is applied to which of the following

appliances?

1. Ballast

2. Domestic Gas Stove

3. Distribution Transformer

4. Agricultural Pumpset

Select the correct answer using the code given below

(a) 1 and 2 only

(b) 1, 3 and 4 only

(c) 2, 3 and 4 only

(d) 1, 2, 3 and 4

Solution: D

The Standards & Labeling Programme is one of the major thrust areas of BEE. A key

objective of this scheme is to provide the consumer an informed choice about the energy

saving and thereby the cost saving potential of the relevant marketed product. The scheme

targets display of energy performance labels on high energy end use equipment &

appliances and lays down minimum energy performance standards.

BEE star labelling is currently invoked for equipments/appliances Room Air Conditioner

(Fixed Speed), Ceiling Fan, Colour Television, Computer, Direct Cool Refrigerator,

Distribution Transformer, Domestic Gas Stove, Frost Free Refrigerator, General

Purpose Industrial Motor, Monoset Pump, Openwell Submersible Pump Set, Stationary

Type Water Heater, Submersible Pump Set, Tfl, Washing Machine, Ballast, Solid State

Inverter, Office Automation Products, Diesel Engine Driven Monosetpumps For

Agricultural Purposes, Diesel Generator Set, Led Lamps, Room Air Conditioner

(Variable Speed), Chillers, Agricultural Pumpset.

https://beeindia.gov.in/content/standards-labeling

https://beestarlabel.com/

2. Consider the following statements regarding Mission Innovation (MI)

1. Mission Innovation (MI) is a global initiative of 24 countries and the

European Commission.

2. MI was announced at COP21, 2015 and India is a member to this initiative.

3. Its aim is to reinvigorate and accelerate global clean energy innovation with

the objective to make clean energy widely affordable.

Page 4: SIMPLYFYING IAS EXAM PREPARATION - INSIGHTSIAS · 2020-04-13 · Insta 75 Days Revision Plan for UPSC Civil Services Prelims – 2020 This document is the compilation of 100 questions

INSTA 75 Days REVISION PLAN for Prelims 2020 - InstaTests

www.insightsonindia.com 2 Insights IAS

Which of the statements given above is/are correct?

(a) 1 and 2 only

(b) 2 and 3 only

(c) 1 and 3 only

(d) 1, 2 and 3

Solution: D

Mission Innovation (MI) is a global initiative of 24 countries and the European

Commission (on behalf of the European Union) working to reinvigorate and accelerate

global clean energy innovation with the objective to make clean energy widely affordable.

• MI was announced at COP21 on November 30, 2015, as world leaders came

together in Paris to commit to ambitious efforts to combat climate change.

• India is a member to this initiative.

As part of the launch statement, members committed to:

• Seek to double their governmental and/or state-directed clean energy clean energy

research, development and demonstration (RD&D) investments over five years.

• Work closely with the private sector as it increases its investment in the earlier-

stage clean energy companies that emerge from government programs.

• Build and improve technology innovation roadmaps and other tools to help in our

innovation efforts, to understand where RD&D is already happening, and to

identify gaps and opportunities for new kinds of innovation.

• Provide, on an annual basis, transparent, easily-accessible information on their

respective clean energy RD&D efforts

Extra Learning:

The Clean Energy Ministerial (CEM) is a high-level global forum to share lessons learnt

and best practices, and to encourage the transition to a global clean energy economy.

Initiatives are based on areas of common interest among participating Governments and

other stakeholders. The CEM is the initiative of the USA. Presently, 23 countries are

members of CEM. There are 13 initiatives under CEM covering energy efficiency, clean

energy supply and cross cutting areas.

Since its launch in 2010, the Clean Energy Ministerial (CEM) has come a long way. Its

various initiatives were instrumental in leading significant collaborative work amongst

members in context of clean energy supply, demand side management and relevant cross

cutting issues.

Page 5: SIMPLYFYING IAS EXAM PREPARATION - INSIGHTSIAS · 2020-04-13 · Insta 75 Days Revision Plan for UPSC Civil Services Prelims – 2020 This document is the compilation of 100 questions

INSTA 75 Days REVISION PLAN for Prelims 2020 - InstaTests

www.insightsonindia.com 3 Insights IAS

3. Consider the following statements regarding World Travel and Tourism Council

(WTTC)

1. WTTC works to raise awareness of Travel and Tourism as one of the

world’s largest economic sectors

2. It is an intergovernmental organization representing the Travel and Tourism

sector globally.

3. Its members include CEOs, Chairpersons and Presidents of the world’s

leading Travel and Tourism companies.

Which of the statements given above is/are correct?

(a) 1 only

(b) 2 and 3 only

(c) 1 and 3 only

(d) 1, 2 and 3

Solution: C

The World Travel & Tourism Council (WTTC) represents the Travel & Tourism private

sector globally. Our Members include over 200 CEOs, Chairpersons and Presidents of the

world’s leading Travel & Tourism companies from all geographies covering all industries.

WTTC works to raise awareness of Travel & Tourism as one of the world’s largest

economic sectors, supporting one in 10 jobs (319 million) worldwide, and generating

10.4% of global GDP.

For the past 30 years, WTTC has conducted research on the economic impact of Travel

& Tourism in 185 countries. In 2018, the Travel & Tourism sector experienced 3.9%

growth, outpacing that of the global economy (3.2%) for the eighth consecutive year. Over

the past five years, one in five jobs were created by the sector, making Travel & Tourism

the best partner for governments to generate employment.

4. Which of the following methods is/are used to effectively manage solid waste

management?

1. Incineration

2. Gasification

3. Pyrolysis

4. Biomethanation

Select the correct answer using the code given below

(a) 2 and 3 only

(b) 1 and 3 only

(c) 1, 2 and 3 only

Page 6: SIMPLYFYING IAS EXAM PREPARATION - INSIGHTSIAS · 2020-04-13 · Insta 75 Days Revision Plan for UPSC Civil Services Prelims – 2020 This document is the compilation of 100 questions

INSTA 75 Days REVISION PLAN for Prelims 2020 - InstaTests

www.insightsonindia.com 4 Insights IAS

(d) 1, 2, 3 and 4

Solution: D

Incineration is a waste treatment process that involves the combustion of organic

substances contained in waste materials. Incineration and other high-temperature waste

treatment systems are described as “thermal treatment”. Incineration of waste materials

converts the waste into ash, flue gas and heat.

Gasification is a process that converts organic or fossil fuel based carbonaceous

materials into carbon monoxide, hydrogen and carbon dioxide. This is achieved by

reacting the material at high temperatures (>700’C), without combustion, with a

controlled amount of oxygen and/or steam(The syngas produced by gasification can be

turned into higher value commercial products)

Pyrolysis involves application of heat with no added oxygen in order to generate oils

and/or syngas (as well as solid waste outputs) and requires more homogenous waste

streams.

Biomethanation is a process by which organic material is microbiologically converted

under anaerobic conditions to biogas. It involves fermenting bacteria, organic acid

oxidizing bacteria, and methanogenic archaea

5. Consider the following statements regarding The Australia Group (AG)

1. It is a legally binding forum of countries which, through the harmonization

of export controls, seeks to ensure that exports do not contribute to the

development of chemical or biological weapons.

2. All states participating in the Australia Group are parties to the Chemical

Weapons Convention (CWC) and the Biological Weapons Convention

(BWC).

Which of the statements given above is/are correct?

(a) 1 only

(b) 2 only

(c) Both 1 and 2

(d) Neither 1 nor 2

Solution: B

Page 7: SIMPLYFYING IAS EXAM PREPARATION - INSIGHTSIAS · 2020-04-13 · Insta 75 Days Revision Plan for UPSC Civil Services Prelims – 2020 This document is the compilation of 100 questions

INSTA 75 Days REVISION PLAN for Prelims 2020 - InstaTests

www.insightsonindia.com 5 Insights IAS

The Australia Group (AG) is an informal forum of countries which, through the

harmonisation of export controls, seeks to ensure that exports do not contribute to the

development of chemical or biological weapons.

Participants in the Australia Group do not undertake any legally binding obligations: the

effectiveness of their cooperation depends solely on a shared commitment to CBW non-

proliferation goals and the strength of their respective national measures.

All states participating in the Australia Group are parties to the Chemical Weapons

Convention (CWC) and the Biological Weapons Convention (BWC), and strongly

support efforts under those Conventions to rid the world of CBW.

Extra Learning:

Missile Technology Control Regime:

Page 8: SIMPLYFYING IAS EXAM PREPARATION - INSIGHTSIAS · 2020-04-13 · Insta 75 Days Revision Plan for UPSC Civil Services Prelims – 2020 This document is the compilation of 100 questions

INSTA 75 Days REVISION PLAN for Prelims 2020 - InstaTests

www.insightsonindia.com 6 Insights IAS

Established in 1987, MTCR is an “informal political understanding among states that seek

to limit the proliferation of missiles and missile technology”. It has 35 members. India was

admitted in June 2016.

China is not a member. MTCR’s initial aim of controlling proliferation of nuclear missiles

was expanded in 1992 to include delivery systems for chemical and biological weapons as

well. It encourages members not to export missiles delivering any weapon of mass

destruction, with special focus on missiles capable of carrying a 500-kg payload over at

least 300 km, and on equipment, software, technology for such systems.

Wassenaar Arrangement

• Named after a suburb in The Hague where it was first agreed upon in 1995, the

Wassenaar Arrangement came into existence a year later. It aims to promote

“transparency and greater responsibility in transfers of conventional arms and dual-

use goods and technologies”, so there are no “destabilising accumulations”, and

terrorists do not acquire them. It has 42 members; India was admitted as a

“participating state” on December 8, 2017. Members must be

producers/exporters of arms/sensitive industrial equipment; have national polices

for non-proliferation and an effective export control regime; must adhere to global

non-proliferation compacts including the Nuclear Non-Proliferation Treaty,

Biological Weapons Convention, Chemical Weapons Convention.

Page 9: SIMPLYFYING IAS EXAM PREPARATION - INSIGHTSIAS · 2020-04-13 · Insta 75 Days Revision Plan for UPSC Civil Services Prelims – 2020 This document is the compilation of 100 questions

INSTA 75 Days REVISION PLAN for Prelims 2020 - InstaTests

www.insightsonindia.com 7 Insights IAS

6. Global Social Mobility Report is released by

(a) World Economic Forum

(b) World Tourism Organization

(c) World Bank

(d) International Organization for Migration (IOM)

Solution: A

Global Social Mobility Report

• World Economic Forum has come out with its first-ever Global Social Mobility

Report.

• The report has ranked India a lowly 76 out of the 82 countries profiled.

What is social mobility?

• It can be understood as the movement in personal circumstances either “upwards”

or “downwards” of an individual in relation to those of their parents.

• In absolute terms, it is the ability of a child to experience a better life than their

parents.

• On the other hand, relative social mobility is an assessment of the impact of socio-

economic background on an individual’s outcomes in life.

• It can be measured against a number of outcomes ranging from health to

educational achievement and income.

7. Consider the following statements regarding National Agricultural Higher

Education Project (NAHEP)

1. It is launched by NITI Ayog to attract talent and strengthen higher

agricultural education in the country

2. It aims to support participating agricultural universities and ICAR in

providing more relevant and higher quality education to Agricultural

University students.

Which of the statements given above is/are correct?

(a) 1 only

(b) 2 only

(c) Both 1 and 2

(d) Neither 1 nor 2

Page 10: SIMPLYFYING IAS EXAM PREPARATION - INSIGHTSIAS · 2020-04-13 · Insta 75 Days Revision Plan for UPSC Civil Services Prelims – 2020 This document is the compilation of 100 questions

INSTA 75 Days REVISION PLAN for Prelims 2020 - InstaTests

www.insightsonindia.com 8 Insights IAS

Solution: B

National Agricultural Higher Education Project (NAHEP).

• Aim: To attract talent and strengthen higher agricultural education in the

country.

• Funded by the World Bank and the Indian Government on a 50:50 basis.

• The objective of the NAHEP for India is to support participating agricultural

universities and ICAR in providing more relevant and higher quality education to

Agricultural University students. In addition, a four year degree in Agriculture,

Horticulture, Fisheries and Forestry has been declared a professional degree.

https://www.insightsonindia.com/2019/01/30/national-agricultural-higher-education-

project-nahep/

8. Disinvestment in a public sector undertaking’s is approved by which of the

following organisation/body?

(a) Parliament

(b) Cabinet Committee on Economic Affairs

(c) NITI Ayog

(d) None of the above

Solution: B

Cabinet Committee on Economic Affairs approves disinvestment in public sector.

The Cabinet Committee on Economic Affairs has approved five PSUs – BPCL, Shipping

Corporation of India, Container Corporation, Tehri Hydro Development Corporation

and North Eastern Electric Power Corporation – for strategic sales.

https://www.thehindubusinessline.com/opinion/quick-take/psu-disinvestment-

strategically-right/article30046722.ece#

Extra Reading:

What is Disinvestment

• Disinvestment in India meaning: Disinvestment means sale or liquidation of

assets by the government, usually Central and state public sector enterprises,

projects, or other fixed assets.

Page 11: SIMPLYFYING IAS EXAM PREPARATION - INSIGHTSIAS · 2020-04-13 · Insta 75 Days Revision Plan for UPSC Civil Services Prelims – 2020 This document is the compilation of 100 questions

INSTA 75 Days REVISION PLAN for Prelims 2020 - InstaTests

www.insightsonindia.com 9 Insights IAS

9. Consider the following statements regarding Pradhan Mantri Kisan Maan-Dhan

Yojana (PM-KMY)

1. It is an old age pension scheme for all land holding Small and Marginal

Farmers (SMFs) in the country.

2. It is a voluntary and contributory pension scheme for the entry age group of

18 to 40 years.

3. The spouse is also eligible to get a separate pension of Rs.3000/- without

making separate contributions to the Fund.

Which of the statements given above is/are correct?

(a) 1 and 2 only

(b) 2 and 3 only

(c) 1 and 3 only

(d) 1, 2 and 3

Solution: A

Pradhan Mantri Kisan Maan-Dhan Yojana (PM-KMY)

• Pradhan Mantri Kisan Maan-Dhan Yojana (PM-KMY) is an old age pension

scheme for all land holding Small and Marginal Farmers (SMFs) in the country.

It is a voluntary and contributory pension scheme for the entry age group of 18 to

40 years. The Scheme is effective from the 9thAugust, 2019.

Salient features

• It is voluntary and contributory for farmers in the entry age group of 18 to 40 years

and a monthly pension of Rs. 3000/- will be provided to them on attaining the age

of 60 years.

• The farmers will have to make a monthly contribution of Rs.55 to Rs.200,

depending on their age of entry, in the Pension Fund till they reach the retirement

date i.e. the age of 60 years. The farmers will have to make a monthly contribution

of Rs.55 to Rs.200, depending on their age of entry, in the Pension Fund till they

reach the retirement date i.e. the age of 60 years.

• The monthly contributions will fall due on the same day every month as enrolment

date. The beneficiaries may also chose an option to pay their contributions on

quarterly, 4-monthly or half-yearly basis. Such contributions will fall due on the

same day of such period as the date of enrollment

• The spouse is also eligible to get a separate pension of Rs.3000/- upon making

separate contributions to the Fund.

• The Life Insurance Corporation of India (LIC) shall be the Pension Fund

Manager and responsible for Pension pay out.

Page 12: SIMPLYFYING IAS EXAM PREPARATION - INSIGHTSIAS · 2020-04-13 · Insta 75 Days Revision Plan for UPSC Civil Services Prelims – 2020 This document is the compilation of 100 questions

INSTA 75 Days REVISION PLAN for Prelims 2020 - InstaTests

www.insightsonindia.com 10 Insights IAS

10. Consider the following statements regarding National Bench of the Goods and

Services Tax Appellate Tribunal

1. It is a quasi-judicial body

2. It is the forum of second appeal in GST lawsand the first common forum of

dispute resolution between Centre and States.

Which of the statements given above is/are correct?

(a) 1 only

(b) 2 only

(c) Both 1 and 2

(d) Neither 1 nor 2

Solution: C

National Bench of the Goods and Services Tax Appellate Tribunal

• It is a quasi-judicial body

• It shall be situated at New Delhi.

• Composition: Presided over by its President and shall consist of one Technical

Member (Centre) and one Technical Member (State).

• It is the forum of second appeal in GST laws and the first common forum of dispute

resolution between Centre and States.

• The appeals against the orders in first appeals issued by the Appellate Authorities

under the Central and State GST Acts lie before the GST Appellate Tribunal,

which is common under the Central as well as State GST Acts.

Legal provisions:

• CGST Act provides for the Appeal and Review Mechanism for dispute resolution

under the GST Regime.

• The Act empowers the Central Government to constitute, on the recommendation

of Council, by notification, with effect from such date as may be specified therein,

an Appellate Tribunal known as the Goods and Services Tax Appellate Tribunal

for hearing appeals against the orders passed by the Appellate Authority or the

Revisional Authority.

https://www.insightsonindia.com/2019/01/24/national-bench-of-the-goods-and-

services-tax-appellate-tribunal-gstat/

11. Consider the following statements regarding SANKALP

Page 13: SIMPLYFYING IAS EXAM PREPARATION - INSIGHTSIAS · 2020-04-13 · Insta 75 Days Revision Plan for UPSC Civil Services Prelims – 2020 This document is the compilation of 100 questions

INSTA 75 Days REVISION PLAN for Prelims 2020 - InstaTests

www.insightsonindia.com 11 Insights IAS

1. It is an outcome-oriented central sector scheme of Ministry of Skill

Development & Entrepreneurship (MSDE).

2. It focuses on the overall skilling ecosystem covering both Central & State

agencies and implements the mandate of the National Skill Development

Mission.

Which of the statements given above is/are correct?

(a) 1 only

(b) 2 only

(c) Both 1 and 2

(d) Neither 1 nor 2

Solution: B

SANKALP is an outcome-oriented centrally sponsored programme of Ministry of Skill

Development & Entrepreneurship (MSDE) with a special focus on decentralised

planning and quality improvement. It focuses on the overall skilling ecosystem covering

both Central & State agencies. SANKALP aims to implement the mandate of the

National Skill Development Mission (NSDM).Under SANKALP four key result areas

have been identified viz: (i) Institutional Strengthening (at National, State & District level);

(ii) Quality Assurance Quality Assurance of skill development programs; (iii) Inclusion of

marginalised population in skill development; and (iv) Expanding Skills through Public

Private Partnerships (PPPs).

12. NetSCoFAN is an initiative for

(a) Reduce post-harvest loss.

(b) Improve productivity in fisheries sector

(c) Export of processed foods

(d) Food safety

Solution: D

Dr Harsh Vardhan launched NetSCoFAN (Network for Scientific Co-operation for

Food Safety and Applied Nutrition), a network of research & academic institutions

working in the area of food & nutrition along with the NetSCoFAN directory, covering

detailed information of various heads/Directors and lead scientists of lead and associated

partnering institutions. The NetSCoFAN would comprise of eight groups of institutions

working in different areas viz. biological, chemical, nutrition & labelling, food of animal

Page 14: SIMPLYFYING IAS EXAM PREPARATION - INSIGHTSIAS · 2020-04-13 · Insta 75 Days Revision Plan for UPSC Civil Services Prelims – 2020 This document is the compilation of 100 questions

INSTA 75 Days REVISION PLAN for Prelims 2020 - InstaTests

www.insightsonindia.com 12 Insights IAS

origin, food of plant origin, water & beverages, food testing, and safer & sustainable

packaging. FSSAI has identified eight Nodal Institutions who would develop a ‘Ready

Reckoner’ that will have inventory of all research work, experts and institutions and would

carry out and facilitate research, survey and related activities. It would identify research

gaps in respective areas and collect, collate and develop database on food safety issues for

risk assessment activities. “The need for identify research gaps in respective areas and

collect, collate and develop database on food safety issues for risk assessment activities,

will be addressed by NetSCoFAN (Network for Scientific Co-operation for Food Safety

and Applied Nutrition)”, Dr Harsh Vardhan said.

13. Consider the following statements regarding Outer Space Treaty

1. It bars states party to the treaty from placing weapons of mass destruction

in orbit of Earth, installing them on the Moon or any other celestial body.

2. The treaty prohibits the placement of conventional weapons in orbit.

3. India is party to the convention.

Which of the statements given above is/are correct?

(a) 1 and 3 only

(b) 2 only

(c) 2 and 3 only

(d) 1 and 2 only

Solution: A

Outer Space Treaty is a legal entity and came in 1966 as a legal document towards the

General Assembly. The Treaty was opened for signature by the three depository

Governments (the Russian Federation, the United Kingdom and the United States of

America) in January 1967, and it entered into force in October 1967.

The Outer Space Treaty represents the basic legal framework of international space

law. Among its principles, it bars states party to the treaty from placing weapons of mass

destruction in orbit of Earth, installing them on the Moon or any other celestial body, or

otherwise stationing them in outer space.

It exclusively limits the use of the Moon and other celestial bodies to peaceful purposes

and expressly prohibits their use for testing weapons of any kind, conducting military man

oeuvres, or establishing military bases, installations, and fortifications (Article IV).

However, the Treaty does not prohibit the placement of conventional weapons in orbit.

The treaty also states that the exploration of outer space shall be done to benefit all

countries and shall be free for exploration and use by all the States.

India is party to Outer Space Treaty.

Page 15: SIMPLYFYING IAS EXAM PREPARATION - INSIGHTSIAS · 2020-04-13 · Insta 75 Days Revision Plan for UPSC Civil Services Prelims – 2020 This document is the compilation of 100 questions

INSTA 75 Days REVISION PLAN for Prelims 2020 - InstaTests

www.insightsonindia.com 13 Insights IAS

14. Consider the following statements regarding TechSagar

1. It is a national repository of India’s cyber tech capabilities.

2. It was launched by National Informatics Centre.

Which of the statements given above is/are correct?

(a) 1 only

(b) 2 only

(c) Both 1 and 2

(d) Neither 1 nor 2

Solution: A

TechSagar

• TechSagar, national repository of India’s cyber tech capabilities launched.

• Launched by: National Cyber Security Coordinator’s office in partnership with

Data Security Council (DSCI) of India.

What is TechSagar?

• It is a platform to discover India’s technological capability through a portal.

• It is a consolidated and comprehensive repository of India’s cyber tech

capabilities which provides actionable insights about capabilities of the Indian

Industry, academia and research across 25 technology areas like internet of things

(IoT), Artificial Intelligence (AI), Machine Learning (ML), blockchain, cloud

& virtualisation, robotics & automation, ar/vr, wireless & networking, and

more.

• The portal will list business and research entities from the IT industry, startups,

academia, and individual researchers.

Extra Learning:

About DSCI:

• Data Security Council of India (DSCI), is a not-for-profit, industry body on data

protection in India, setup by NASSCOM.

• It is committed to making the cyberspace safe, secure and trusted by establishing

best practices, standards and initiatives in cyber security and privacy.

• To further its objectives, DSCI engages with governments and their agencies,

regulators, industry sectors, industry associations and think tanks for policy

advocacy, thought leadership, capacity building and outreach activities.

Page 16: SIMPLYFYING IAS EXAM PREPARATION - INSIGHTSIAS · 2020-04-13 · Insta 75 Days Revision Plan for UPSC Civil Services Prelims – 2020 This document is the compilation of 100 questions

INSTA 75 Days REVISION PLAN for Prelims 2020 - InstaTests

www.insightsonindia.com 14 Insights IAS

15. ‘SWISS Challenge Model’, often in news, is

(a) One of the method of awarding government contracts to private players.

(b) Operation to uncover black money from Switzerland.

(c) Challenge to clean corrupt practices in taxation system in India

(d) None of the above

Solution: A

Swiss Challenge method is one of the ways of awarding government contracts to

private players. Without an invitation from government, a private player can submit a

proposal to government for development of an infrastructure project with exclusive

intellectual property rights.

https://www.thehindubusinessline.com/opinion/columns/slate/all-you-wanted-to-

know-about-swiss-challenge/article24194034.ece

16. Consider the following statements regarding Exchange Traded Funds (ETFs)

1. These are listed and traded in the stock exchange like shares.

2. It contains different types of investments including stocks, commodities, or

bonds.

3. ETF is managed by the investor himself unless deputed.

Which of the statements given above is/are correct?

(a) 2 and 3 only

(b) 1 and 3 only

(c) 1 and 2 only

(d) 1, 2 and 3

Solution: D

• The centre is planning to launch India’s first fixed income Exchange Traded Fund

(ETF) comprising debt securities of large central public sector enterprises (PSUs).

What are ETFs?

• Exchange Traded Funds (ETFs) are mutual funds listed and traded on stock

exchanges like shares.

Page 17: SIMPLYFYING IAS EXAM PREPARATION - INSIGHTSIAS · 2020-04-13 · Insta 75 Days Revision Plan for UPSC Civil Services Prelims – 2020 This document is the compilation of 100 questions

INSTA 75 Days REVISION PLAN for Prelims 2020 - InstaTests

www.insightsonindia.com 15 Insights IAS

• Typically, an ETF mirrors a particular index, which means the group of stocks in

the ETF would be similar to those in the index that it is benchmarked to.

• Usually, ETFs are passive funds where the fund manager doesn’t select stocks on

your behalf. Instead, the ETF simply copies an index and endeavours to accurately

reflect its performance.

• In an ETF, one can buy and sell units at prevailing market price on a real time basis

during market hours.

• ETFs can contain all types of investments including stocks, commodities, or bonds;

some offer domestic holdings only, while others are international.

Benefits and significance of ETFs:

• ETFs are cost efficient. Given that they don’t make any stock (or security choices),

they don’t use services of star fund managers.

• They allow investors to avoid the risk of poor security selection by the fund

manager, while offering a diversified investment portfolio.

• The stocks in the indices are carefully selected by index providers and are

rebalanced periodically.

• They offer anytime liquidity through the exchanges.

• Mutual fund is like an ETF as it is a unit that comprised of equities of different

companies. But ETF and mutual funds differ with respect to tradability. Mutual

Fund selling price will be the price of shares at the close of the day. On the other

hand, shares of ETF are traded throughout the day. And at any moment, ETF can

be bought and sold. In this respect, ETFs have more liquidity and marketability.

Another difference is that Mutual Fund is managed by a financial company and its

fund managers; whereas the ETF is managed by the investor himself (unless

deputed).

https://www.insightsonindia.com/2019/11/25/exchange-traded-funds-2/

17. Consider the following statements regarding National Service Scheme (NSS)

1. NSS is an extension dimension to the higher education system to orient the

student youth to community service while they are studying in educational

institutions.

2. It is being implemented by the Ministry of Youth Affairs and Sports.

Which of the statements given above is/are correct?

(a) 1 only

(b) 2 only

(c) Both 1 and 2

(d) Neither 1 nor 2

Solution: C

Page 18: SIMPLYFYING IAS EXAM PREPARATION - INSIGHTSIAS · 2020-04-13 · Insta 75 Days Revision Plan for UPSC Civil Services Prelims – 2020 This document is the compilation of 100 questions

INSTA 75 Days REVISION PLAN for Prelims 2020 - InstaTests

www.insightsonindia.com 16 Insights IAS

National Service Scheme (NSS) was launched during 1969, the birth centenary year of

Mahatma Gandhi, in 37 universities involving 40000 students. NSS is an extension

dimension to the higher education system to orient the student youth to community service

while they are studying in educational institutions. It is being implemented by the Ministry

of Youth Affairs and Sports, Government of India.

• NSS is a voluntary scheme. NSS scheme starts from 11th Standard onwards. At

college level the NSS volunteers will be enrolled from the first- and second-year

degree-class students. The students from foreign countries studying in Indian

universities can also join NSS so that they may share the experiences of national

reconstruction and community work. NCC cadets will not be allowed to join NSS.

Similarly NSS volunteers will not participate in NCC or any other youth

organisation as long as they are in NSS.

• An NSS volunteer needs to devote a total of 240 hours social service in two years

duration. Every year, a NSS volunteer has to devote 20 hrs. of orientation and 100

hrs. of community service.

• To enrol as NSS volunteer, contact the NSS Programme Officer of your school /

college. Enrollment in NSS is free.

• The NSS volunteers on successful completion of the required hours of service are

issued a certificate. NSS volunteers can get some weightage during admissions in

higher studies and other benefits as decided by the institutions/university.

18. Project SU.RE is a commitment in

(a) Solar energy

(b) Apparel industry

(c) Electronic products

(d) Unorganized sector

Solution: B

The Union Minister for Textiles, Smt. Smriti Zubin Irani launched Project SU.RE, a

move towards sustainable fashion, at Lakmé Fashion Winter/Festive 2019, in Mumbai

on 22nd August, 2019. The SU.RE project is a commitment by India’s apparel industry to

set a sustainable pathway for the Indian fashion industry. SU.RE stands for ‘Sustainable

Resolution’ – a firm commitment from the industry to move towards fashion that

contributes to a clean environment.

• The project has been launched by the Minister, along with Clothing

Manufacturers Association of India (CMAI); United Nations in India; and IMG

Reliance, the organizers of Lakmé Fashion Week.

Page 19: SIMPLYFYING IAS EXAM PREPARATION - INSIGHTSIAS · 2020-04-13 · Insta 75 Days Revision Plan for UPSC Civil Services Prelims – 2020 This document is the compilation of 100 questions

INSTA 75 Days REVISION PLAN for Prelims 2020 - InstaTests

www.insightsonindia.com 17 Insights IAS

About Project SU.RE

The five-point Sustainable Resolution is as follows:

1. Develop a complete understanding of the environmental impact of the garments

being currently produced by our brand.

2. Develop a sustainable sourcing policy for consistently prioritizing and utilizing

certified raw materials that have a positive impact on the environment.

3. Make the right decisions about how, where, and what we source across the value

chain by selecting sustainable and renewable materials and processes and ensuring

their traceability.

4. Communicate our sustainability initiatives effectively to consumers and media

through our online and physical stores, product tags/labeling, social media,

advertising campaigns and events.

5. Through these actions, shift a significant percentage of our supply chain to a

sustainable chain by the year 2025, addressing critical global issues such as climate

change, contributing to the UN Sustainable Development Goals, and building a

world that is safe for the future generations, as an acceptance of a responsibility we

all share.

Project SU.RE, driven by the leading industry association CMAI, along with a leader in

sustainable fashion IMG Reliance, will be the first holistic effort by the apparel industry

towards gradually introducing a broader framework for establishing critical sustainability

goals for the industry. This framework would help the industry reduce its carbon

emissions, increase resource efficiency, tackle waste and water management, and create

positive social impact to achieve long-term sustainability targets.

19. Consider the following statements regarding Surakshit Matritva Aashwasan

1. It aims to provide dignified and quality health care at no cost to every

woman and newborn visiting a public health facility.

2. Under the scheme, all pregnant women, newborns and mothers up to 6

months of delivery will be able to avail free health care services.

Which of the statements given above is/are correct?

(a) 1 only

(b) 2 only

(c) Both 1 and 2

(d) Neither 1 nor 2

Solution: C

Page 20: SIMPLYFYING IAS EXAM PREPARATION - INSIGHTSIAS · 2020-04-13 · Insta 75 Days Revision Plan for UPSC Civil Services Prelims – 2020 This document is the compilation of 100 questions

INSTA 75 Days REVISION PLAN for Prelims 2020 - InstaTests

www.insightsonindia.com 18 Insights IAS

The Union Government has launched Surakshit Matritva Aashwasan (SUMAN) to

provide quality healthcare at zero cost to pregnant women, new mothers and

newborns.

About the scheme:

• It aims to provide dignified and quality health care at no cost to every woman and

newborn visiting a public health facility.

• Under the scheme, the beneficiaries visiting public health facilities are entitled to

several free services.

• These include at least four ante natal check-ups that also includes one checkup

during the 1st trimester, at least one checkup under Pradhan Mantri Surakshit

Matritva Abhiyan, Iron Folic Acid supplementation, Tetanus diptheria

injection.

• Eligibility: All pregnant women, newborns and mothers up to 6 months of

delivery will be able to avail several free health care services

Features and benefits:

• The scheme will enable zero expense access to the identification and management

of complications during and after the pregnancy.

• The government will also provide free transport to pregnant women from home to

the health facility and drop back after discharge (minimum 48 hrs).

• The pregnant women will be able to avail a zero expense delivery and C-section

facility in case of complications at public health facilities.

• The scheme will ensure that there is zero-tolerance for denial of services to such

patients.

https://www.insightsonindia.com/2019/10/11/surakshit-matritva-aashwasan-suman/

20. Consider the following statements regarding National e-Vidhan Application

(NeVA) Project

1. It is a Mission Mode Project (MMP) under the Digital India Programme.

2. Ministry of Parliamentary Affairs (MoPA) is the Nodal Ministry for its

implementation in all the States/UTs with Legislatures.

Which of the statements given above is/are correct?

(a) 1 only

(b) 2 only

(c) Both 1 and 2

(d) Neither 1 nor 2

Solution: C

Page 21: SIMPLYFYING IAS EXAM PREPARATION - INSIGHTSIAS · 2020-04-13 · Insta 75 Days Revision Plan for UPSC Civil Services Prelims – 2020 This document is the compilation of 100 questions

INSTA 75 Days REVISION PLAN for Prelims 2020 - InstaTests

www.insightsonindia.com 19 Insights IAS

National e-Vidhan Application (NeVA) Project

• The Kerala Legislative Assembly recently announced an initiative to digitize all

its records and proceedings under its ambitious project called E-Vidhan.

What is e-Vidhan?

• It is a Mission Mode Project (MMP) comes under the Digital India Programme.

• Ministry of Parliamentary Affairs (MoPA) is the ‘Nodal Ministry’ for its

implementation in all the States/UTs with Legislatures.

• The funding for e-Vidhan is provided by the MoPA and technical support by

Ministry of Electronics and Information Technology (MietY).

• The funding of NeVA is through Central Sponsored Scheme. 60:40; and 90:10 for

North East & hilly States and 100% for UTs.

• Aim of the project: To bring all the legislatures of the country together, in one

platform thereby creating a massive data depository without having the complexity

of multiple applications.

21. Consider the following statements regarding Central Vigilance Commission (CVC)

1. The Commission was set up on the recommendation of the K.Santhanam

Committee

2. Term of the central vigilance commissioner is 5 years or 60 years, whichever

is earlier.

3. It submits its report to the President of India.

Which of the statements given above is/are correct?

(a) 1 only

(b) 2 and 3 only

(c) 1 and 3 only

(d) 1, 2 and 3

Solution: C

Vigilance Awareness Week to be observed from 28th October to 2nd November. Theme

of the Vigilance Awareness Week: “Integrity- A way of life”

Background:

• The Central Vigilance Commission aims to promote integrity, transparency and

accountability in public life.

• As part of its efforts to promote probity in public life and to achieve a corruption

free society, CVC observes Vigilance Awareness Week every year.

Page 22: SIMPLYFYING IAS EXAM PREPARATION - INSIGHTSIAS · 2020-04-13 · Insta 75 Days Revision Plan for UPSC Civil Services Prelims – 2020 This document is the compilation of 100 questions

INSTA 75 Days REVISION PLAN for Prelims 2020 - InstaTests

www.insightsonindia.com 20 Insights IAS

• The observation of Vigilance Awareness Week creates greater awareness among

public and encourages all the stakeholders to collectively participate in prevention

of and fight against Corruption.

About CVC:

• It is the apex vigilance institution created via executive resolution (based on the

recommendations of Santhanam committee) in 1964 but was conferred with

statutory status in 2003.

• It submits its report to the President of India.

• Composition: Consists of central vigilance commissioner along with 2 vigilance

commissioners.

• Appointment: They are appointed by the President of India on the

recommendations of a committee consisting of Prime Minister, Union Home

Minister and Leader of the Opposition in Lok Sabha (if there is no LoP then the

leader of the single largest Opposition party in the Lok Sabha).

• Term: Their term is 4 years or 65 years, whichever is earlier.

• Removal: The Central Vigilance Commissioner or any Vigilance Commissioner

can be removed from his office only by order of the President on the ground of

proved misbehavior or incapacity after the Supreme Court, on a reference made to

it by the President, has, on inquiry, reported that the Central Vigilance

Commissioner or any Vigilance Commissioner, as the case may be, ought to be

removed.

https://www.insightsonindia.com/2019/10/26/insights-daily-current-affairs-pib-26-

october-2019/

22. Consider the following statements regarding Rashtriya Sanskriti Mahotsav

1. It was organized by Ministry of Human Resource Development under the

Ek Bharat Shrestha Bharat initiative.

2. It was organized to showcase the rich cultural heritage of the Country in all

its rich and varied dimensions.

Which of the statements given above is/are correct?

(a) 1 only

(b) 2 only

(c) Both 1 and 2

(d) Neither 1 nor 2

Solution: B

Page 23: SIMPLYFYING IAS EXAM PREPARATION - INSIGHTSIAS · 2020-04-13 · Insta 75 Days Revision Plan for UPSC Civil Services Prelims – 2020 This document is the compilation of 100 questions

INSTA 75 Days REVISION PLAN for Prelims 2020 - InstaTests

www.insightsonindia.com 21 Insights IAS

Ministry of Culture, Government of India is organising the 10th edition of Rashtriya

Sanskriti Mahotsav under the Ek Bharat Shrestha Bharat initiative in Madhya Pradesh

from 14th to 21st October 2019.

Being the flagship and premier festival of Ministry of Culture, Rashtriya Sanskriti

Mahotsav was conceived in the year 2015 and after the grand success of the First Rashtriya

Sanskriti Mahotsav in November-2015, the Ministry of Culture decided to organize it with

an intent to showcase the rich cultural heritage of the Country in all its rich and varied

dimensions, viz Handicrafts, Cuisine, Painting, Sculpture and Performing Arts-Folk,

Tribal, Classical and Contemporary- all in one place. So far, this Ministry has organised 9

RSM’s i.e. 2 each in Delhi and 2 in Karnataka, Uttar Pradesh, RSM North East , Gujarat,

Madhya Pradesh and Uttarakahnd.The people of Madhya Pradesh will witness the

grandeur of Rashtriya Sanskriti Mahotsav once again in its 10th edition from 14th

October, 2019 to 21st October, 2019.

This time the Mahotsav will be organised in the city of Jabalpur during 14 – 15 October,

2019,Sagar from 16 – 17th October, 19 and Rewa from 20 – 21st October, 2019.

The Ek Bharat Shreshtha Bharat programme was launched by the Prime Minister on 31st

October, 2016 to promote engagement amongst the people of different States/UTs so as

to enhance mutual understanding and bonding between people of diverse cultures, thereby

securing stronger unity and integrity of India.

23. Consider the following statements regarding District Mineral Foundation

1. These are non-statutory bodies set by the executive resolution.

2. Each District Mineral Foundation is established by the Central government

by notification.

3. The Pradhan Mantri Khanij Kshetra Kalyan Yojana uses the funds

generated by the District Mineral Foundations.

Which of the statements given above is/are correct?

(a) 1 and 3 only

(b) 2 and 3 only

(c) 3 only

(d) 1, 2 and 3

Solution: C

Rajasthan to create pneumoconiosis fund with DMF money. The fund will be used to

execute a comprehensive policy on the disease, which is widely prevalent in the mining

state.

Page 24: SIMPLYFYING IAS EXAM PREPARATION - INSIGHTSIAS · 2020-04-13 · Insta 75 Days Revision Plan for UPSC Civil Services Prelims – 2020 This document is the compilation of 100 questions

INSTA 75 Days REVISION PLAN for Prelims 2020 - InstaTests

www.insightsonindia.com 22 Insights IAS

Background:

• Pneumoconiosis, a lung disease, mostly affects workers who work in the mining

and construction sectors and deal with soil, silica, coal dust and asbestos. The

disease includes asbestosis, silicosis and coal workers’ pneumoconiosis.

About DMFs:

• District Mineral Foundations are statutory bodies set up as non-profit

organizations in districts which are affected by the mining works.

• Each District Mineral Foundation is established by the State Governments by

notification as a trust or non-profit body in the mining operation affected districts.

• DMFs were instituted under the Mines and Minerals (Development and

Regulation) (MMDR) Amendment Act 2015.

• They are non-profit trusts to work for the interest and benefit of persons and areas

affected by mining-related operations.

• Objective: to work for the interest of the benefit of the persons and areas affected

mining related operations in such manner as may be prescribed by the State

Government.

• Jurisdiction: Its manner of operation comes under the jurisdiction of the relevant

State Government.

The various state DMF rules and the Pradhan Mantri Khanij Khestra Kalyan Yojana

(PMKKKY) guidelines stipulate some “high priority” issues for DMFs, including:

1. Drinking water.

2. Health

3. Women and child welfare.

4. Education

5. Livelihood and skill development.

6. Welfare of aged and disabled.

7. Sanitation.

Pradhan Mantri Khanij Kshetra Kalyan Yojana (PMKKKY):

• The programme is meant to provide for the welfare of areas and people affected by

mining related operations, using the funds generated by District Mineral

Foundations (DMFs).

Objectives of the scheme:

• To implement various developmental and welfare projects/programs in mining

affected areas that complement the existing ongoing schemes/projects of State and

Central Government.

• To minimize/mitigate the adverse impacts, during and after mining, on the

environment, health and socio-economics of people in mining districts.

• To ensure long-term sustainable livelihoods for the affected people in mining areas.

https://www.insightsonindia.com/2019/10/04/district-mineral-foundations-4/

Page 25: SIMPLYFYING IAS EXAM PREPARATION - INSIGHTSIAS · 2020-04-13 · Insta 75 Days Revision Plan for UPSC Civil Services Prelims – 2020 This document is the compilation of 100 questions

INSTA 75 Days REVISION PLAN for Prelims 2020 - InstaTests

www.insightsonindia.com 23 Insights IAS

24. Consider the following statements regarding Samudrayaan project

1. It is a pilot project of the Ministry of Earth Sciences for deep ocean mining

for rare minerals.

2. The project has been undertaken by the National Institute of Ocean

Technology (NIOT).

Which of the statements given above is/are correct?

(a) 1 only

(b) 2 only

(c) Both 1 and 2

(d) Neither 1 nor 2

Solution: C

‘Samudrayaan’ project

• India to undertake deep ocean mining with ‘Samudrayaan’ project.

About Samudrayaan:

• It is a pilot project of the Ministry of Earth Sciences for deep ocean mining for rare

minerals.

• It proposes to send men into the deep sea in a submersible vehicle for ocean studies.

• The project is expected to become a reality by 2021-22.

• The project has been undertaken by the National Institute of Ocean Technology

(NIOT).

What are PMN?

• Polymetallic nodules (also known as manganese nodules) are potato- shaped,

largely porous nodules found in abundance carpeting the sea floor of world oceans

in deep sea.

• Composition: Besides manganese and iron, they contain nickel, copper, cobalt,

lead, molybdenum, cadmium, vanadium, titanium, of which nickel, cobalt and

copper are considered to be of economic and strategic importance.

• Potential: It is envisaged that 10% of recovery of that large reserve can meet the

energy requirement of India for the next 100 years. It has been estimated that 380

million metric tonnes of polymetallic nodules are available at the bottom of the seas

in the Central Indian Ocean.

Page 26: SIMPLYFYING IAS EXAM PREPARATION - INSIGHTSIAS · 2020-04-13 · Insta 75 Days Revision Plan for UPSC Civil Services Prelims – 2020 This document is the compilation of 100 questions

INSTA 75 Days REVISION PLAN for Prelims 2020 - InstaTests

www.insightsonindia.com 24 Insights IAS

25. Consider the following statements regarding National Health Systems Resource

Centre (NHSRC)

1. It is a five member governing body.

2. It has been set up under the National Rural Health Mission (NRHM)

3. It aims to assist in policy and strategy development in the provision and

mobilization of technical assistance to the states and centre.

Which of the statements given above is/are correct?

(a) 3 only

(b) 1 and 3 only

(c) 2 and 3 only

(d) 1, 2 and 3

Solution: C

Designation of National Health Systems Resource Centre (NHSRC) as a WHO

Collaborating Centre for Priority Medical Devices and Health Technology Policy.

Why do we need such collaborations?

• Such global collaborations in the area of health technology will ensure that

scientific and technological advances, research and development as well as

innovative technologies play a substantial supportive role in healthcare and enable

us to reach the public health goals and achieve universal health coverage.

About NHSRC:

• Set up under the National Rural Health Mission (NRHM) to serve as an apex

body for technical assistance.

• Established in 2006.

• Mandate is to assist in policy and strategy development in the provision and

mobilization of technical assistance to the states and in capacity building for the

Ministry of Health and Family Welfare (MoHFW) at the centre and in the states.

Composition:

• It has a 23 member Governing Body, chaired by the Secretary, MoHFW,

Government of India with the Mission Director, NRHM as the Vice Chairperson

of the GB and the Chairperson of its Executive Committee.

• Of the 23 members, 14 are ex-officio senior health administrators, including four

from the states. Nine are public health experts, from academics and Management

Experts.

• The Executive Director, NHSRC is the Member Secretary of both the Governing

body and the Executive Committee.

Page 27: SIMPLYFYING IAS EXAM PREPARATION - INSIGHTSIAS · 2020-04-13 · Insta 75 Days Revision Plan for UPSC Civil Services Prelims – 2020 This document is the compilation of 100 questions

INSTA 75 Days REVISION PLAN for Prelims 2020 - InstaTests

www.insightsonindia.com 25 Insights IAS

• The NHSRC currently consists of seven divisions – Community Processes,

Healthcare Financing, Healthcare Technology, Human Resources for Health,

Public Health Administration, Public Health Planning, Quality Improvement in

Healthcare.

https://www.insightsonindia.com/2019/10/04/national-health-systems-resource-

centre-nhsrc/

DAY – 22 (InstaTest-22)

26. Sepahijala wildlife sanctuary, sometime seen in the news, is located in which of the

following state?

(a) Assam

(b) Tripura

(c) West Bengal

(d) Sikkim

Solution: B

Sepahijala Wildlife Sanctuary is a wildlife sanctuary in Tripura, India of some 18.53

square kilometres, about 25 kilometres from the city centre, located in Bishalgarh. It is a

woodland with an artificial lake and natural botanical and zoological gardens. It is

famous for its clouded leopard enclosures.

https://indianexpress.com/article/north-east-india/tripura/tripuras-best-kept-secret-

three-lion-cubs-born-in-june-5517621/

27. Oslo accords, often seen in the news, is related to

(a) Reduction of Ozone depleting gases

(b) Refugee crisis

(c) Israeli-Palestinian Peace plan

(d) Syrian civil war

Solution: C

Page 28: SIMPLYFYING IAS EXAM PREPARATION - INSIGHTSIAS · 2020-04-13 · Insta 75 Days Revision Plan for UPSC Civil Services Prelims – 2020 This document is the compilation of 100 questions

INSTA 75 Days REVISION PLAN for Prelims 2020 - InstaTests

www.insightsonindia.com 26 Insights IAS

Oslo Accords

• Palestinians had threatened to quit Oslo Accords, if U.S. President Donald Trump

announces his West Asia peace plan.

Why?

Palestinians’ concerns:

• The main worry is that this initiative will turn Israel’s “temporary occupation (of

Palestinian territory) into a permanent occupation”.

• The Palestinians see east Jerusalem as the capital of their future state and believe

Mr. Trump’s plan buries the two-state solution that has been for decades the

cornerstone of international West Asia diplomacy.

What’s the issue?

• Under the Oslo Accords of the 1993, both Israel and the Palestinians agreed that

the status of settlements would be decided by negotiations. But the negotiations

process has been all but dead for several years now.

• Israel walked into East Jerusalem in 1967, and subsequently annexed it. For Israel,

Jerusalem is non-negotiable.

• The Palestinians want East Jerusalem as the capital of their future state.

• Most of the world’s nations look at it as occupied territory.

About the Oslo Accords:

• Formally known as the Declaration of Principles (DOP), the pact established a

timetable for the Middle East peace process. It planned for an interim Palestinian

government in Gaza and Jericho in the West Bank.

• Oslo II, officially called the Israeli-Palestinian Interim Agreement on the West

Bank and Gaza, expanded on Oslo I. It included provisions for the complete

withdrawal of Israeli troops from six West Bank cities and about 450 towns.

Additionally, the pact set a timetable for elections for the Palestinian Legislative

Council.

28. Consider the following statements regarding East Asia Summit

1. It is an initiative of ASEAN and is based on the premise of the centrality of

ASEAN.

2. It is a forum for strategic dialogue and cooperation on political, security and

economic issues.

3. India has been participating in the EAS since its very inception in 2005.

Which of the statements given above is/are correct?

(a) 1 and 2 only

(b) 2 and 3 only

(c) 1 and 3 only

Page 29: SIMPLYFYING IAS EXAM PREPARATION - INSIGHTSIAS · 2020-04-13 · Insta 75 Days Revision Plan for UPSC Civil Services Prelims – 2020 This document is the compilation of 100 questions

INSTA 75 Days REVISION PLAN for Prelims 2020 - InstaTests

www.insightsonindia.com 27 Insights IAS

(d) 1, 2 and 3

Solution: D

East Asia Summit:

• India hosted an East Asia Summit conference in Chennai with a focus on maritime

security cooperation and tackling challenges in the maritime domain.

• It was organized by the Ministry of External Affairs (MEA), in partnership with

the governments of Australia and Indonesia.

• This conference is the fourth in a series of EAS Maritime Security Conferences

organized by the Indian government.

About East Asia Summit:

• EAS is an initiative of ASEAN and is based on the premise of the centrality of

ASEAN.

• It is a forum held annually by leaders of 18 countries in the East Asian, Southeast

Asian and South Asian regions.

• EAS meetings are held after annual ASEAN leaders’ meetings.

• The first summit was held in Kuala Lumpur, Malaysia on 14 December 2005.

• India has been participating in the EAS since its very inception in 2005.

• 14th summit was held in November 2019 in Thailand.

• EAS has evolved as a forum for strategic dialogue and cooperation on political,

security and economic issues of common regional concern and plays an important

role in the regional architecture.

• There are six priority areas of regional cooperation within the framework of the

EAS. These are – Environment and Energy, Education, Finance, Global Health

Issues and Pandemic Diseases, Natural Disaster Management, and ASEAN

Connectivity. India endorses regional collaboration in all six priority areas.

• EAS is a region of strong and fast-growing economies. It is considered the third

pole of world economy after the US and Europe.

29. Consider the following statements regarding Web- Wonder Women Campaign

1. It is a campaign launched by CARE India

2. It aims to provide assistance to underprivileged women & girls in rural

areas.

Which of the statements given above is/are correct?

(a) 1 only

(b) 2 only

(c) Both 1 and 2

Page 30: SIMPLYFYING IAS EXAM PREPARATION - INSIGHTSIAS · 2020-04-13 · Insta 75 Days Revision Plan for UPSC Civil Services Prelims – 2020 This document is the compilation of 100 questions

INSTA 75 Days REVISION PLAN for Prelims 2020 - InstaTests

www.insightsonindia.com 28 Insights IAS

(d) Neither 1 nor 2

Solution: D

The Ministry of Women and Child Development, Government of India, has launched

an online campaign, ‘#www : Web- Wonder Women’.

The Campaign aims to discover and celebrate the exceptional achievements of women,

who have been driving positive agenda of social change via social media.

Through the campaign, the Ministry and the Campaign’s Partners aim to recognize the

fortitude of Indian women stalwarts from across the globe who have used the power of

social media to run positive & niche campaigns to steer a change in society. This

Campaign will recognize and acknowledge the efforts of these meritorious Women.

https://pib.gov.in/Pressreleaseshare.aspx?PRID=1559269

30. Consider the following statements regarding European Union

1. The EU and European citizenship were established when the Maastricht

Treaty came into force.

2. The United Kingdom became the first member state ever to leave the EU.

Which of the statements given above is/are correct?

(a) 1 only

(b) 2 only

(c) Both 1 and 2

(d) Neither 1 nor 2

Solution: C

European Union

• United Kingdom has officially left the European Union (EU) and has become the

first country to leave the 28-member bloc. The UK joined in 1973 (when it was

known as the European Economic Community) and it will be the first member

state to withdraw.

What is the European Union?

• The EU is an economic and political union involving 27 European countries. It

allows free trade, which means goods can move between member countries without

Page 31: SIMPLYFYING IAS EXAM PREPARATION - INSIGHTSIAS · 2020-04-13 · Insta 75 Days Revision Plan for UPSC Civil Services Prelims – 2020 This document is the compilation of 100 questions

INSTA 75 Days REVISION PLAN for Prelims 2020 - InstaTests

www.insightsonindia.com 29 Insights IAS

any checks or extra charges. The EU also allows free movement of people, to live

and work in whichever country they choose.

• The EU and European citizenship were established when the Maastricht Treaty

came into force in 1993.

• The EU traces its origins to the European Coal and Steel Community (ECSC) and

the European Economic Community (EEC), established, respectively, by the 1951

Treaty of Paris and 1957 Treaty of Rome.

• The latest major amendment to the constitutional basis of the EU, the Treaty of

Lisbon, came into force in 2009.

• In January 2020, the United Kingdom became the first member state ever to leave

the EU.

• The union represents itself at the United Nations, the World Trade Organization,

the G7 and the G20.

31. Consider the following statements regarding International Atomic Energy Agency

(IAEA)

1. The Agency works to promote the safe, secure and peaceful use of nuclear

technologies.

2. It is an organ of the United Nations and reports to both the United Nations

General Assembly and Security Council.

Which of the statements given above is/are correct?

(a) 1 only

(b) 2 only

(c) Both 1 and 2

(d) Neither 1 nor 2

Solution: A

Widely known as the world’s “Atoms for Peace and Development” organization within

the United Nations family, the IAEA is the international centre for cooperation in the

nuclear field. The Agency works with its Member States and multiple partners worldwide

to promote the safe, secure and peaceful use of nuclear technologies.

• The IAEA was created in 1957 in response to the deep fears and expectations

generated by the discoveries and diverse uses of nuclear technology. The Agency’s

genesis was U.S. President Eisenhower’s “Atoms for Peace” address to the

General Assembly of the United Nations on 8 December 1953.

Page 32: SIMPLYFYING IAS EXAM PREPARATION - INSIGHTSIAS · 2020-04-13 · Insta 75 Days Revision Plan for UPSC Civil Services Prelims – 2020 This document is the compilation of 100 questions

INSTA 75 Days REVISION PLAN for Prelims 2020 - InstaTests

www.insightsonindia.com 30 Insights IAS

International Atomic Energy Agency (IAEA)

• The IAEA is the world’s centre for cooperation in the nuclear field.

• It was set up as the world’s “Atoms for Peace” organization in 1957.

• It was established as an autonomous organization in 1957 through its own

international treaty, the IAEA Statute.

• The Agency works with its Member States and multiple partners worldwide to

promote the safe, secure and peaceful use of nuclear technologies.

• It seeks to promote the peaceful use of nuclear energy, and to inhibit its use for any

military purpose, including nuclear weapons.

• IAEA reports to both the United Nations General Assembly and Security

Council.

• The IAEA has its headquarters in Vienna, Austria.

• The IAEA serves as an intergovernmental forum for scientific and technical

cooperation in the peaceful use of nuclear technology and nuclear power

worldwide.

• The IAEA and its former Director General, Mohamed ElBaradei, were jointly

awarded the Nobel Peace Prize on 7 October 2005.

• The IAEA was established as an autonomous organisation on 29 July 1957.

Though established independently of the United Nations through its own

international treaty, the IAEA Statute, the IAEA reports to both the United

Nations General Assembly and Security Council.

32. Consider the following statements regarding Mycobacterium Indicus Pranii (MIP)

1. It is an indigenous vaccine for multi-drug resistance Tuberculosis

2. It was developed by the National Institute of Immunology, New Delhi

Which of the statements given above is/are correct?

(a) 1 only

(b) 2 only

(c) Both 1 and 2

(d) Neither 1 nor 2

Solution: B

It is an indigenous vaccine for leprosy developed by National Institute of Immunology

(New Delhi).

• It is now being introduced into the National Leprosy Elimination Programme

(NLEP). It will boost the immune system against the bacterial disease

Page 33: SIMPLYFYING IAS EXAM PREPARATION - INSIGHTSIAS · 2020-04-13 · Insta 75 Days Revision Plan for UPSC Civil Services Prelims – 2020 This document is the compilation of 100 questions

INSTA 75 Days REVISION PLAN for Prelims 2020 - InstaTests

www.insightsonindia.com 31 Insights IAS

About Leprosy:

• The disease, caused by the bacteria, Mycobacterium leprae, affects around 1.25

lakh people every year in India. Sixty per cent of the world’s leprosy patients live

in India.

• The vaccine, called Mycobacterium indicus pranii (MIP), will be administered as

a preventive measure to people living in close contact with those infected. It was

developed by the National Institute of Immunology, New Delhi, and has been by

approved by the Drug Controller General of India and the FDA in the U.S.

https://www.thehindu.com/sci-tech/health/medicine-and-research/Made-in-India-

leprosy-vaccine-to-be-launched/article14581750.ece

33. Consider the following statements regarding Stockholm International Peace

Research Institute (SIPRI)

1. It is an intergovernmental organization based in Stockholm.

2. The institute is dedicated to research into conflict, armaments, arms control

and disarmament.

Which of the statements given above is/are correct?

(a) 1 only

(b) 2 only

(c) Both 1 and 2

(d) Neither 1 nor 2

Solution: B

Stockholm International Peace Research Institute (SIPRI)

• SIPRI is an independent international institute dedicated to research into conflict,

armaments, arms control and disarmament. Established in 1966, SIPRI provides

data, analysis and recommendations, based on open sources, to policymakers,

researchers, media and the interested public. Based in Stockholm, SIPRI is

regularly ranked among the most respected think tanks worldwide.

Vision and mission:

• SIPRI’s vision is a world in which sources of insecurity are identified and

understood, conflicts are prevented or resolved, and peace is sustained.

SIPRI’s mission is to:

• undertake research and activities on security, conflict and peace;

• provide policy analysis and recommendations;

Page 34: SIMPLYFYING IAS EXAM PREPARATION - INSIGHTSIAS · 2020-04-13 · Insta 75 Days Revision Plan for UPSC Civil Services Prelims – 2020 This document is the compilation of 100 questions

INSTA 75 Days REVISION PLAN for Prelims 2020 - InstaTests

www.insightsonindia.com 32 Insights IAS

• facilitate dialogue and build capacities;

• promote transparency and accountability; and

• deliver authoritative information to global audiences.

Statutes:

• The Statutes are the guiding principles of SIPRI’s work. They were adopted by the

Swedish Government.

History and funding

• SIPRI was established on the basis of a decision by the Swedish Parliament and

receives a substantial part of its funding in the form of an annual grant from the

Swedish Government. The Institute also seeks financial support from other

organizations in order to carry out its research.

34. Consider the following statements regarding Asian Development Bank (ADB)

1. The bank was proposed by China and based in Beijing.

2. It has more than hundred (100) approved members.

Which of the statements given above is/are correct?

(a) 1 only

(b) 2 only

(c) Both 1 and 2

(d) Neither 1 nor 2

Solution: D

The Asian Infrastructure Investment Bank (AIIB) was proposed by China in 2013 and it

is seen as a potential rival to the World Bank and IMF.

• The Asian Infrastructure Investment Bank (AIIB) is a multilateral development

bank with a mission to improve social and economic outcomes in Asia.

Headquartered in Beijing, we began operations in January 2016 and have now

grown to 102 approved members worldwide. By investing in sustainable

infrastructure and other productive sectors in Asia and beyond, we will better

connect people, services and markets that over time will impact the lives of billions

and build a better future.

The Asian Development Bank (ADB) envisions a prosperous, inclusive, resilient, and

sustainable Asia and the Pacific, while sustaining its efforts to eradicate extreme poverty

in the region. Despite the region’s many successes, it remains home to a large share of the

world’s poor: 264 million living on less than $1.90 a day and 1.1 billion on less than $3.20

a day.

Page 35: SIMPLYFYING IAS EXAM PREPARATION - INSIGHTSIAS · 2020-04-13 · Insta 75 Days Revision Plan for UPSC Civil Services Prelims – 2020 This document is the compilation of 100 questions

INSTA 75 Days REVISION PLAN for Prelims 2020 - InstaTests

www.insightsonindia.com 33 Insights IAS

• ADB assists its members, and partners, by providing loans, technical assistance,

grants, and equity investments to promote social and economic development.

• ADB maximizes the development impact of its assistance by facilitating policy

dialogues, providing advisory services, and mobilizing financial resources through

co-financing operations that tap official, commercial, and export credit sources.

• ADB was conceived in the early 1960s as a financial institution that would be Asian

in character and foster economic growth and cooperation in one of the poorest

regions in the world.

A resolution passed at the first Ministerial Conference on Asian Economic Cooperation

held by the United Nations Economic Commission for Asia and the Far East in 1963 set

that vision on the way to becoming reality.

• The Philippines capital of Manila was chosen to host the new institution, which

opened on 19 December 1966, with 31 members that came together to serve a

predominantly agricultural region. Takeshi Watanabe was ADB’s first President.

• From 31 members at its establishment in 1966, ADB has grown to encompass 68

members—of which 49 are from within Asia and the Pacific and 19 outside.

https://www.adb.org/about/members

35. Consider the following statements regarding the Land for Life Programme

1. The Land for Life Programme was launched at the tenth United Nations

Convention to Combat Desertification Conference of the Parties (CoP 10)

2. It is involved in a variety of awareness-raising and knowledge support

activities to provide information and sensitize the public about the

importance of land for their life.

Which of the statements given above is/are correct?

(a) 1 only

(b) 2 only

(c) Both 1 and 2

(d) Neither 1 nor 2

Solution: C

The Land for Life Programme was launched at the tenth UNCCD Conference of the

Parties (COP10) in 2011 in the Republic of Korea as part of the Changwon Initiative.

The Programme seeks to address the challenges of land degradation, desertification and

mitigation of drought.

Page 36: SIMPLYFYING IAS EXAM PREPARATION - INSIGHTSIAS · 2020-04-13 · Insta 75 Days Revision Plan for UPSC Civil Services Prelims – 2020 This document is the compilation of 100 questions

INSTA 75 Days REVISION PLAN for Prelims 2020 - InstaTests

www.insightsonindia.com 34 Insights IAS

• The Land for Life Programme is involved in a variety of awareness-raising and

knowledge support activities to provide information and sensitize the public about

the importance of land for their life and for achieving the SDGs by 2030.

These activities include:

• Promoting LDN

• Highlighting SLM success stories and the achievements of Land for Life Award

winners

• Promoting public outreach activities such as:

• Producing educational videos

• Holding photo contests

• Sharing and disseminating knowledge

• Conducting media trainings

• Organizing campaigns and e-Forums

https://www.unccd.int/actions/land-life

36. Consider the following statements regarding SUTRA PIC initiative

1. The programme was launched by Ministry of Fisheries, Animal Husbandry

and Dairying.

2. It is a programme to research on indigenous cows.

Which of the statements given above is/are correct?

(a) 1 only

(b) 2 only

(c) Both 1 and 2

(d) Neither 1 nor 2

Solution: B

SUTRA PIC

• The government has unveiled a programme to research on ‘indigenous’ cows-

SUTRA PIC.

About SUTRA PIC- Scientific Utilisation Through Research Augmentation-Prime

Products from Indigenous Cows:

• It is led by the Department of Science and Technology (DST).

• It is supported by the Department of Biotechnology, the Council of Scientific and

Industrial Research, the Ministry for AYUSH (Ayurveda, Unani, Siddha,

Page 37: SIMPLYFYING IAS EXAM PREPARATION - INSIGHTSIAS · 2020-04-13 · Insta 75 Days Revision Plan for UPSC Civil Services Prelims – 2020 This document is the compilation of 100 questions

INSTA 75 Days REVISION PLAN for Prelims 2020 - InstaTests

www.insightsonindia.com 35 Insights IAS

Homoeopathy) among others and the Indian Council of Medical Research as

partners.

It has five themes:

1. Uniqueness of Indigenous Cows.

2. Prime-products from Indigenous Cows for Medicine and Health.

3. Prime-products from Indigenous Cows for Agricultural Applications.

4. Prime-products from Indigenous Cows for Food and Nutrition.

5. Prime-products from indigenous cows-based utility items.

Aims and objectives:

• Scientific research on the complete characterization of milk and milk products

derived from Indian indigenous cows.

• Scientific research on nutritional and therapeutic properties of curd and ghee

prepared from indigenous breeds of cows by traditional methods.

• Development of standards for traditionally processed dairy products of Indian-

origin cows, etc.

37. Consider the following statements regarding Common Services Centres (CSCs)

1. The CSC was initiated under Digital India Initiative in 2016.

2. The Scheme creates a conducive environment for the private sector and

NGOs to play an active role in implementation of the CSC Scheme.

3. CSC e-Governance Services India Limited, a Special Purpose Vehicle, has

been set up to oversee implementation of the CSC scheme.

Which of the statements given above is/are correct?

(a) 1 and 2 only

(b) 2 and 3 only

(c) 1 and 3 only

(d) 1, 2 and 3

Solution: B

Common Services Centres

The CSC is a strategic cornerstone of the National e-Governance Plan (NeGP), approved

by the Government in May 2006, as part of its commitment in the National Common

Minimum Programme to introduce e-governance on a massive scale.

The CSCs would provide high quality and cost-effective video, voice and data content and

services, in the areas of e-governance, education, health, telemedicine, entertainment as

well as other private services. A highlight of the CSCs is that it will offer web-enabled e-

Page 38: SIMPLYFYING IAS EXAM PREPARATION - INSIGHTSIAS · 2020-04-13 · Insta 75 Days Revision Plan for UPSC Civil Services Prelims – 2020 This document is the compilation of 100 questions

INSTA 75 Days REVISION PLAN for Prelims 2020 - InstaTests

www.insightsonindia.com 36 Insights IAS

governance services in rural areas, including application forms, certificates, and utility

payments such as electricity, telephone and water bills. In addition to the universe of G2C

services, the CSC Guidelines envisage a wide variety of content and services that could be

offered as listed below:

1. Agriculture Services (Agriculture, Horticulture, Sericulture, Animal Husbandry,

Fisheries, Veterinary)

2. Education & Training Services (School, College, Vocational Education,

Employment, etc.)

3. Health Services (Telemedicine, Health Check-ups, Medicines)

4. Rural Banking & Insurance Services (Micro-credit, Loans, Insurance)

5. Entertainment Services (Movies, Television)

6. Utility Services (Bill Payments, Online bookings)

7. Commercial Services (DTP, Printing, Internet Browsing, Village level BPO).

The Scheme creates a conducive environment for the private sector and NGOs to play an

active role in implementation of the CSC Scheme, thereby becoming a partner of the

government in development of rural India. The PPP model of the CSC scheme envisages

a 3-tier structure consisting of the CSC operator (called Village Level Entrepreneur or

VLE); the Service Centre Agency (SCA), that will be responsible for a division of 500-1000

CSCs; and a State Designated Agency (SDA) identified by the State Government

responsible for managing the implementation in the entire State.

CSC e-Governance Services India Limited, a Special Purpose Vehicle, has been set up by

the Ministry of Electronics & IT under the Companies Act, 1956 to oversee

implementation of the CSC scheme. CSC SPV provides a centralized collaborative

framework for delivery of services to citizens through CSCs, besides ensuring systemic

viability and sustainability of the Scheme.

38. Which of the following species is/are naturally found in India?

1. Nilgai

2. Spider Monkey

3. Spectacled Monkey

4. Striped Hyenas

Select the correct answer using the code given below

(a) 1 and 3 only

(b) 2 and 4 only

(c) 1, 3 and 4 only

(d) 1, 2, 3 and 4

Solution: C

Page 39: SIMPLYFYING IAS EXAM PREPARATION - INSIGHTSIAS · 2020-04-13 · Insta 75 Days Revision Plan for UPSC Civil Services Prelims – 2020 This document is the compilation of 100 questions

INSTA 75 Days REVISION PLAN for Prelims 2020 - InstaTests

www.insightsonindia.com 37 Insights IAS

The Nilgai (Boselaphus tragocamelus) is an antelope found in Asia. It commonly seen in

India, Nepal and eastern Pakistan. The mature males appear ox-like and are also known

as Blue bulls. The nilgai is the biggest Asian antelope.

• Spider monkeys are New World monkeys belonging to the genus Ateles, part of

the subfamily Atelinae, family Atelidae. Like other atelines, they are found in

tropical forests of Central and South America, from southern Mexico to Brazil.

• The Phayre’s leaf monkey (Trachypithecus phayrei), also known as Spectacled

Monkey, is a species of lutung native to Southeast Asia including India,

Bangladesh, Myanmar, Thailand, Laos, Vietnam and China. It is listed as

Endangered on the IUCN Red List and is threatened by hunting and loss of habitat

• Striped Hyenas are naturally found in India.

https://www.thehindu.com/sci-tech/energy-and-environment/an-initiative-to-save-the-

striped-hyenas-of-india/article26446129.ece

39. Consider the following statements regarding Pradhan Mantri Bhartiya

Janaushadhi Pariyojana

1. It was initiated by Ministry of Health and Family Welfare.

2. Bureau of Pharma PSUs of India (BPPI) is the implementing agency of

Pradhan Mantri Janaushadhi Pariyojana (PMBJP).

Which of the statements given above is/are correct?

(a) 1 only

(b) 2 only

(c) Both 1 and 2

(d) Neither 1 nor 2

Solution: B

“Pradhan Mantri Bhartiya Janaushadhi Pariyojana” is a noble initiative by Department

of Pharmaceuticals, Government of India which is now making an impact on masses in

its endeavor to provide quality medicines at an affordable price. The number of Kendras

has grown to more than 6200 and 700 districts are covered under the scheme at present.

Further, total sales in the financial year 2019-20 (up to February 2020) have crossed Rs

383 crore which led to total savings of approximately Rs 2200 crore of common citizens

as these medicines are cheaper by 50 to 90% of the average market price. The scheme is

also providing a good source of self-employment with sustainable and regular earnings.

Bureau of Pharma PSUs of India (BPPI) is the implementing agency of Pradhan Mantri

Janaushadhi Pariyojana (PMBJP). BPPI was established in December, 2008 under the

Department of Pharmaceuticals, Government of India. The Bureau has been registered as

Page 40: SIMPLYFYING IAS EXAM PREPARATION - INSIGHTSIAS · 2020-04-13 · Insta 75 Days Revision Plan for UPSC Civil Services Prelims – 2020 This document is the compilation of 100 questions

INSTA 75 Days REVISION PLAN for Prelims 2020 - InstaTests

www.insightsonindia.com 38 Insights IAS

an independent society under the Societies Registration Act, 1860 as a separate

independent legal entity in April, 2010.

40. Consider the following statements regarding Atal Bimit Vyakti Kalyan Yojana

1. The scheme was initiated by ESI Corporation.

2. Under the scheme, the Insured Person (IP) (who is rendered unemployed)

will be provided relief to the extent of 25% of the average per day earning,

up to maximum 90 days of unemployment.

Which of the statements given above is/are correct?

(a) 1 only

(b) 2 only

(c) Both 1 and 2

(d) Neither 1 nor 2

Solution: C

The ESI Corporation has launched a scheme named ‘Atal Bimit Vyakti Kalyan Yojana’

(ABVKY) which, in case the Insured Person (IP) is rendered unemployed, provides relief

to the extent of 25% of the average per day earning during the previous four contribution

periods (total earning during the four contribution period/730) to be paid up to maximum

90 days of unemployment once in lifetime of the IP on submission of claim in form of an

Affidavit.

Atal Bimit Vyakti Kalyan Yojana’ (ABVKY)

• The Employee’s State Insurance (ESI) has approved this scheme for Insured

Persons (IP) covered under the Employees’ State Insurance Act, 1948.

• It aims to financially support those who lost their jobs or rendered jobless for

whatsoever reasons due to changing employment pattern.

• Its beneficiaries will be insured persons covered under Employees’ State Insurance

Act, 1948 for period of two years continuously.

Key features:

• Cash assistance: Under the scheme, relief will be payable in cash directly to bank

account of insured persons in case of unemployment. This financial assistance will

be given to insured persons even while they search for new engagement.

Beneficiary insured workers will be paid money, from their own contribution

towards ESI scheme, in cash through bank account transfer.

• Under this scheme, workers will be able to draw 47% of their total contributions

towards ESIC after remaining unemployed for at least three months from date of

leaving their previous jobs. They can choose to receive the cash at one go or in

Page 41: SIMPLYFYING IAS EXAM PREPARATION - INSIGHTSIAS · 2020-04-13 · Insta 75 Days Revision Plan for UPSC Civil Services Prelims – 2020 This document is the compilation of 100 questions

INSTA 75 Days REVISION PLAN for Prelims 2020 - InstaTests

www.insightsonindia.com 39 Insights IAS

instalments. It will be applicable to all factories and establishments employing at

least 10 workers.

41. Consider the following statements regarding Drake Passage

1. It is located between Cape Horn at the tip of South America and the

Antarctic Peninsula.

2. It connects the south-western part of the Atlantic Ocean with the south-

eastern part of the Pacific Ocean.

Which of the statements given above is/are correct?

(a) 1 only

(b) 2 only

(c) Both 1 and 2

(d) Neither 1 nor 2

Solution: C

Drake Passage:

• It is located between Cape Horn at the tip of South America and the Antarctic

Peninsula.

• It is the shortest crossing from Antarctica to any other landmass. There is no

significant land anywhere around the world at the latitudes of Drake Passage,

which is important to the unimpeded flow of the Antarctic Circumpolar Current

which carries a huge volume of water through the Passage and around Antarctica.

• The passage is named after Sir Francis Drake, who was the first Englishman to

circumnavigate the globe.

• It connects the south-western part of the Atlantic Ocean (Scotia Sea) with the

south-eastern part of the Pacific Ocean and extends into the Southern Ocean.

Page 42: SIMPLYFYING IAS EXAM PREPARATION - INSIGHTSIAS · 2020-04-13 · Insta 75 Days Revision Plan for UPSC Civil Services Prelims – 2020 This document is the compilation of 100 questions

INSTA 75 Days REVISION PLAN for Prelims 2020 - InstaTests

www.insightsonindia.com 40 Insights IAS

https://www.insightsonindia.com/2019/12/30/insights-daily-current-affairs-pib-

summary-30-december-2019/

42. Consider the following statements regarding Youth Co:Lab

1. It was co-created by the United Nations Development Programme (UNDP)

and the Citi Foundation.

2. Youth Co:Lab positions young people to solve the region’s most pressing

challenges.

Which of the statements given above is/are correct?

(a) 1 only

(b) 2 only

(c) Both 1 and 2

(d) Neither 1 nor 2

Solution: C

Youth Co:Lab positions young people front and center in order to solve the region’s most

pressing challenges. Co-created in 2017 by the United Nations Development Programme

(UNDP) and the Citi Foundation, Youth Co:Lab aims to establish a common agenda for

countries in the Asia-Pacific region to empower and invest in youth, so that they can

accelerate the implementation of the Sustainable Development Goals (SDGs) through

leadership, social innovation and entrepreneurship. Youth Co:Lab has been implemented

in 25 countries and territories across Asia Pacific: Australia, Bangladesh, Bhutan,

Cambodia, China, Fiji, Hong Kong (SAR), Indonesia, India, Japan, Malaysia, the

Maldives, Mongolia, Nepal, Pakistan, Philippines, Samoa, Singapore, Solomon Islands,

South Korea, Sri Lanka, Thailand, Timor Leste, Vanuatu and Viet Nam.

• In a latest initiative to recognize young people as critical drivers of sustainable

development, Atal Innovation Mission (AIM), NITI Aayog and United Nations

Development Programme (UNDP) India on Friday launched Youth Co:Lab which

aims at accelerating social entrepreneurship and innovation in young India.

• To mark the launch, a Letter of Intent (LOI) was signed between AIM, NITI

Aayog and UNDP India.

• Through Youth Co:Lab, young entrepreneurs and innovators will get a chance to

connect with governments, mentors, incubators and investors, who will help equip

them with entrepreneurial skills.

• The initiative will also convene a series of youth dialogues across several cities such

as New Delhi, Hyderabad, Bangalore and Mumbai to promote entrepreneurship

across India.

• AIM and UNDP, as part of UNSDF signed between NITI Aayog and UN India,

are collaborating to spread awareness about different issues pertaining to youth, the

Page 43: SIMPLYFYING IAS EXAM PREPARATION - INSIGHTSIAS · 2020-04-13 · Insta 75 Days Revision Plan for UPSC Civil Services Prelims – 2020 This document is the compilation of 100 questions

INSTA 75 Days REVISION PLAN for Prelims 2020 - InstaTests

www.insightsonindia.com 41 Insights IAS

future of work and the Sustainable Development Goals (SDG) through Youth

Co:Lab.

• The first phase of Youth Co:Lab will focus on six SDGs: SDG 5 (Gender Equality),

SDG 6 (Clean Water and Sanitation), SDG 7 (Affordable and Clean Energy), SDG

8 (Decent Work and Economic Growth), SDG 12 (Sustainable Consumption and

Production) and SDG 13 (Climate Action).

43. FutureSkills is an initiative of

(a) NASSCOM

(b) National Skill Development Corporation

(c) Confederation of Indian Industry (CII)

(d) United Nations Industrial Development Organization (UNIDO)

Solution: A

Future Skills:

• As a host of emerging technologies change the future of work, a massive disruption

is facing the IT-ITES industry today. Of the 4.5 m people employed in the industry

today, 1.5 -2 m are expected to require reskilling in the next 4-5 years. A decoupling

of revenue and headcount growth is visible even today and employers and

employees need to adapt themselves to the changing job environment of

technological shifts and changing stakeholder expectations. The problem is too

large to be handled alone. It needs a collaborative industry level response. With

NASSCOM as the enabler, IT-ITeS Industry has stepped up to the challenge with

the FutureSkills Initiative – a truly industry driven learning ecosystem.

• NASSCOM FutureSkills was launched on 19th February 2018 in the presence of

senior industry leaders and government officials. The programme aims to reskill 2

million professionals and potential employees & students in the industry over a

period of 5 years. To do this, FutureSkills portal uses the technology of the future,

to create a space where a learner can access content on all the skills of the future.

A learner can seamlessly access free and paid content, assessments, virtual labs and

get certified on the skills of their choice.

• The platform is fueled by a curation engine that has the ability to trawl the massive

amount of high-quality learning content available on the web and funnel it in an

easy to consume format that’s meaningful and effective for the learners. It also has

a curation feature that allows Subject Matter Experts from Industry and academia

to contribute by curating content and learning pathways on these technologies.

Page 44: SIMPLYFYING IAS EXAM PREPARATION - INSIGHTSIAS · 2020-04-13 · Insta 75 Days Revision Plan for UPSC Civil Services Prelims – 2020 This document is the compilation of 100 questions

INSTA 75 Days REVISION PLAN for Prelims 2020 - InstaTests

www.insightsonindia.com 42 Insights IAS

44. Consider the following statements regarding RISAT-2BR1

1. It is a radar imaging earth observation satellite

2. The life of the mission is 5 years.

3. It will be placed in the Geosynchronous orbit

Which of the statements given above is/are correct?

(a) 1 and 2 only

(b) 2 and 3 only

(c) 1 and 3 only

(d) 1, 2 and 3

Solution: A

• India’s Polar Satellite Launch Vehicle, in its fiftieth flight (PSLV-C48), has

successfully launched RISAT-2BR1 along with nine commercial satellites from the

Satish Dhawan Space Centre (SDSC) SHAR, Sriharikota. This is PSLV’s 50th

successful mission and the 75th launch vehicle mission from SDSC SHAR,

Sriharikota.

What is RISAT-2BR1?

• It is a radar imaging earth observation satellite.

• It provides services in the field of agriculture, forestry and disaster management.

• Its mission life is 5 years.

• It will be placed in the low-earth orbit of 576 km at an inclination of 37 degrees.

Other satellites on board:

• The nine customer satellites were from Israel, Italy, Japan and the USA.

• These satellites were launched under a commercial arrangement with New Space

India Limited (NSIL).

Background:

• The RISAT, which was first deployed in orbit on April 20, 2009 as the RISAT-2,

uses synthetic aperture radars (SAR) to provide Indian forces with all-weather

surveillance and observation, which are crucial to notice any potential threat or

malicious activity around the nation’s borders.

• Following the 2008 Mumbai terror attacks, the launch of RISAT-2 was prioritised

over RISAT- 1, as its C-band SAR radar was not yet ready and RISAT -2 carried

an Israeli-built X-band radar.

https://www.insightsonindia.com/2019/12/12/risat-2br1-2/

Page 45: SIMPLYFYING IAS EXAM PREPARATION - INSIGHTSIAS · 2020-04-13 · Insta 75 Days Revision Plan for UPSC Civil Services Prelims – 2020 This document is the compilation of 100 questions

INSTA 75 Days REVISION PLAN for Prelims 2020 - InstaTests

www.insightsonindia.com 43 Insights IAS

45. Consider the following statements regarding Central Water Commission

1. It functions as an attached office of the Ministry of Jal Shakti.

2. It is a statutory body set up under the Water (Prevention and Control of

Pollution) Act, 1974

Which of the statements given above is/are correct?

(a) 1 only

(b) 2 only

(c) Both 1 and 2

(d) Neither 1 nor 2

Solution: A

Central Water Commission is a premier Technical Organization of India in the field of

Water Resources and is presently functioning as an attached office of the Ministry of Jal

Shakti, Department of Water Resources, River Development and Ganga Rejuvenation,

Government of India.

Page 46: SIMPLYFYING IAS EXAM PREPARATION - INSIGHTSIAS · 2020-04-13 · Insta 75 Days Revision Plan for UPSC Civil Services Prelims – 2020 This document is the compilation of 100 questions

INSTA 75 Days REVISION PLAN for Prelims 2020 - InstaTests

www.insightsonindia.com 44 Insights IAS

• The Commission is entrusted with the general responsibilities of initiating,

coordinating and furthering in consultation of the State Governments concerned,

schemes for control, conservation and utilization of water resources throughout the

country, for purpose of Flood Control, Irrigation, Navigation, Drinking Water

Supply and Water Power Development. It also undertakes the investigations,

construction and execution of any such schemes as required.

History:

• CWC (erstwhile Central Waterways, Irrigation and Navigation Commission) was

established in 1945 by the Government on the advice of Dr. B.R. Ambedkar,

Member (Labour) in Viceroy’s Executive Council. The credit for the establishment

of CWINC is attributed to Dr. B. R. Ambedkar, under whose able guidance the

then Labour Department constituted the Commission. He not only raised the

concept and argued for the necessity of having such a technical body at the Centre

but also laid down its objectives, organisational structure and programme. The final

proposal for establishment of CWINC was prepared by the Department with the

help of Rai Bahadur A.N. Khosla, the Consulting Engineer for Irrigation. Dr.

Khosla was subsequently appointed as founder Chairman of the CWINC.

• Since then CWC has evolved as a premier technical organisation in the country in

the field of Water Resources and now has entered into 75thand to celebrate the

same several seminars, workshops & other activities have been planned throughout

the year. (Thus, important for this year)

• Central Water Commission CWC Central Water Commission CWC is headed by

a Chairman, with the status of Ex-Officio Secretary to the Government of India.

• The work of the Commission is divided among 3 wings namely, Designs and

Research (D&R) Wing, River Management (RM) Wing and Water Planning and

Projects (WP&P) Wing.

46. Consider the following statements regarding National Green Corps

1. It was launched by Ministry of Human Resource Development

2. It aims to provide opportunities for children to understand the environment

and environmental problems through school eco-clubs.

Which of the statements given above is/are correct?

(a) 1 only

(b) 2 only

(c) Both 1 and 2

(d) Neither 1 nor 2

Solution: B

Page 47: SIMPLYFYING IAS EXAM PREPARATION - INSIGHTSIAS · 2020-04-13 · Insta 75 Days Revision Plan for UPSC Civil Services Prelims – 2020 This document is the compilation of 100 questions

INSTA 75 Days REVISION PLAN for Prelims 2020 - InstaTests

www.insightsonindia.com 45 Insights IAS

Launched under the Environment Education Awareness and Training (EEAT), the

National Green Corps (NGC) popularly known as “a programme of Ecoclubs” is a

nationwide initiative of the Ministry of Environment & Forests, Government of India

(now Ministry of Environment, Forests and Climate Change).

Objectives:

• To impart knowledge to school children, through hands-on experience, about their

immediate environment, interactions within it and the problems therein.

• To develop requisite skills of observation, experimentation, survey, recording,

analysis and reasoning for conserving the environment through various activities.

• To inculcate the proper attitude towards the environment and its conservation

through community interactions.

• To sensitize children to issues related to environment and development through

field visits and demonstrations.

• To promote logical and independent thinking among children so that they are able

to make the right choices in a spirit of scientific inquiry.

• To motivate and stimulate young minds by involving them in action projects

related to environmental conservation.

• To provide opportunities for children to understand the environment and

environmental problems through school eco-clubs.

Methodology:

• The scheme is being operated through Eco-clubs of 50-60 students having interest

in environment related issues, formed in member schools.

• Eco clubs are supervised by a Teacher In-charge, who is selected from among the

teachers of the member school.

• There is District Implementation and Monitoring Committee to supervise, organise

training for In-charge teachers, and monitor periodically the implementation of

scheme at the District level.

• There is a State Steering Committee for guidance, direction and to oversee the

implementation of the scheme.

• The State Nodal Agency coordinates the implementation of the scheme in the State

and organize related activities like training to Master Trainers.

• The National Steering Committee will give overall direction to the programme and

ensure linkages at all levels.

https://www.insightsonindia.com/2019/12/24/national-green-corps-ecoclub/

47. Consider the following statements regarding Chief Commissioner for Persons with

Disabilities

1. It was established as non-statutory authority through the executive

resolution.

2. The Chief Commissioner can take suo-moto notice of non-implementation

of any rule, law, etc., meant for persons with disabilities.

Page 48: SIMPLYFYING IAS EXAM PREPARATION - INSIGHTSIAS · 2020-04-13 · Insta 75 Days Revision Plan for UPSC Civil Services Prelims – 2020 This document is the compilation of 100 questions

INSTA 75 Days REVISION PLAN for Prelims 2020 - InstaTests

www.insightsonindia.com 46 Insights IAS

Which of the statements given above is/are correct?

(a) 1 only

(b) 2 only

(c) Both 1 and 2

(d) Neither 1 nor 2

Solution: B

Chief Commissioner for Persons with Disabilities:

• The Chief Commissioner is an important statutory functionary, appointed under

Section 57 of the Persons with Disabilities (Equal Opportunities, Protection of

Rights and Full Participation) Act, 1995. The functions and duties of the Chief

Commissioner include: coordinating the work of state commissioners for persons

with disabilities, monitoring of utilisation of funds disbursed by the central

government, taking steps to safeguard rights and facilities made available to

persons with disabilities and also to look into complaints with respect to

deprivation of rights of persons with disabilities.

• The Chief Commissioner can also take suo moto notice of non-implementation of

any rule, law, etc., meant for persons with disabilities and is vested with the powers

of a civil court relating to summoning of witness, discovery, requistioning and

production of any document, etc.

48. Consider the following statements regarding eBkry portal

1. It has been launched by Reserve Bank of India (RBI)

2. It aims to enable online auction by banks of attached assets transparently

and cleanly for the improved realization of value.

Which of the statements given above is/are correct?

(a) 1 only

(b) 2 only

(c) Both 1 and 2

(d) Neither 1 nor 2

Solution: B

Page 49: SIMPLYFYING IAS EXAM PREPARATION - INSIGHTSIAS · 2020-04-13 · Insta 75 Days Revision Plan for UPSC Civil Services Prelims – 2020 This document is the compilation of 100 questions

INSTA 75 Days REVISION PLAN for Prelims 2020 - InstaTests

www.insightsonindia.com 47 Insights IAS

eBkry portal:

Union Finance Ministry has recently launched the eBkry e-auction portal.

Objective: To enable online auction by banks of attached assets transparently and

cleanly for the improved realization of value.

Key features:

• It is framework for promoting online auction of assets attached by the banks.

• It is equipped with the property search features and contains navigational links to

all PSBs e-auction sites.

• The framework aims to provide single-window access to information on properties.

https://www.insightsonindia.com/2019/12/30/insights-daily-current-affairs-pib-

summary-30-december-2019/

49. Consider the following statements regarding Pradhan Mantri Surakshit Matritva

Abhiyan

1. The program aims to provide assured, comprehensive and quality antenatal

care, free of cost, universally to all pregnant women on the 9th of every

month.

2. It has been launched by the Ministry of Health & Family Welfare

(MoHFW).

3. PMSMA guarantees a minimum package of antenatal care services to

women in their 1st / 2nd trimesters of pregnancy.

Which of the statements given above is/are correct?

(a) 1 only

(b) 2 and 3 only

(c) 1 and 2 only

(d) 1, 2 and 3

Solution: C

The Pradhan Mantri Surakshit Matritva Abhiyan has been launched by the Ministry of

Health & Family Welfare (MoHFW), Government of India. The program aims to

provide assured, comprehensive and quality antenatal care, free of cost, universally to all

pregnant women on the 9th of every month.

Page 50: SIMPLYFYING IAS EXAM PREPARATION - INSIGHTSIAS · 2020-04-13 · Insta 75 Days Revision Plan for UPSC Civil Services Prelims – 2020 This document is the compilation of 100 questions

INSTA 75 Days REVISION PLAN for Prelims 2020 - InstaTests

www.insightsonindia.com 48 Insights IAS

Hon’ble Prime Minister of India highlighted the aim and purpose of introduction of the

Pradhan Mantri Surakshit Matritva Abhiyan in the 31st July 2016 episode of Mann Ki

Baat.

PMSMA guarantees a minimum package of antenatal care services to women in their 2nd

/ 3rd trimesters of pregnancy at designated government health facilities.

The programme follows a systematic approach for engagement with private sector which

includes motivating private practitioners to volunteer for the campaign; developing

strategies for generating awareness and appealing to the private sector to participate in the

Abhiyan at government health facilities.

50. Consider the following statements regarding Satavahanas

1. The last ruler of this dynasty was Gautamiputra Satakarni.

2. They patronized both Buddhism and Brahmanism.

Which of the statements given above is/are correct?

(a) 1 only

(b) 2 only

(c) Both 1 and 2

(d) Neither 1 nor 2

Solution: B

• The Archaeological Survey of India (ASI) has discovered the earliest Sanskrit

inscription in South India. This is also an earliest epigraphic evidence (Epigraphy

is the study of ancient inscriptions) for the Saptamatrika cult.

• The discovery was made in Chebrolu village in Guntur district of Andhra Pradesh.

About the inscriptions found:

• It is in Sanskrit and in Brahmi characters.

• It was issued by Satavahana king Vijaya in 207 A.D.

• The inscription records construction of a prasada (temple), a mandapa (a pavilion

for public rituals) and consecration of images on southern side of temple by a

person named Kartika for merit of king at temple of Bhagavathi (Goddess)

Saktimatruka (Saptamatrika) at Tambrape (which is the ancient name of

Chebrolou).

• Chebrolu inscription of Satavahana king Vijaya issued in his 5th regnal year (207

A.D.) is also the earliest datable Sanskrit inscription from South India so far. Until

now the Nagarjunakonda inscription of Ikshavaku king Ehavala Chantamula

issued in his 11th regnal year (4th century A.D.) was considered the earliest

Sanskrit inscription in South India.

Page 51: SIMPLYFYING IAS EXAM PREPARATION - INSIGHTSIAS · 2020-04-13 · Insta 75 Days Revision Plan for UPSC Civil Services Prelims – 2020 This document is the compilation of 100 questions

INSTA 75 Days REVISION PLAN for Prelims 2020 - InstaTests

www.insightsonindia.com 49 Insights IAS

Satavahanas:

• They are an ancient Indian dynasty based In the Deccan.

• They established their independent rule after the decline of the Mauryas.

• Their rule lasted for about 450 years.

• They were also known as the Andhras.

• The Puranas and the Nasik and Nanaghad inscriptions remain important sources

for the history of Satavahanas.

• The Satavahana kingdom mainly comprised the present-day Telangana, Andhra

Pradesh and Maharashtra. At different times, their rule extended to parts of

modern Gujarat, Madhya Pradesh, and Karnataka. The dynasty had different

capital cities at different times, including Pratishthana (Paithan) and Amaravati

(Dharanikota).

• The founder of the Satavahana dynasty was Simuka.

• The greatest ruler of the Satavahana dynasty was Gautamiputra Satakarni.

• Last great ruler of this dynasty was Yajna sri Satkarni, who recovered Malwa &

northern coast of konkan from Shaka rulers (which was captured by Rudradaman

before)

• They patronized Buddhism and Brahmanism.

https://www.insightsonindia.com/2019/12/28/south-indias-earliest-sanskrit-

inscription-found-in-ap/

DAY – 23 (InstaTest-23)

51. Consider the following statements regarding Eat Right Movement

1. It was launched by the Ministry of Food Processing Industries.

2. It also aims to engage and enable citizens to improve their health and well-

being by making the right food choices.

Which of the statements given above is/are correct?

(a) 1 only

(b) 2 only

(c) Both 1 and 2

(d) Neither 1 nor 2

Solution: B

Western Railway’s Mumbai Central Terminus has become India’s first Eat Right

Station. Food Safety & Standards Authority of India (FSSAI) has rated the station with

four stars.

Page 52: SIMPLYFYING IAS EXAM PREPARATION - INSIGHTSIAS · 2020-04-13 · Insta 75 Days Revision Plan for UPSC Civil Services Prelims – 2020 This document is the compilation of 100 questions

INSTA 75 Days REVISION PLAN for Prelims 2020 - InstaTests

www.insightsonindia.com 50 Insights IAS

Background:

• The station was graded on the basis of food safety and hygiene, availability of

healthy diet, food handling at preparation and food waste management among

others.

• The food was certified and rated after a thorough inspection. ‘Eat Right Station’

is a part of the ‘Eat Right India’ initiative that was launched by FSSAI in 2018 to

help passengers make a healthy choice.

About Eat Right Movement:

• It was launched by the Food Safety and Standards Authority of India (FSSAI).

• The movement aims to cut down salt/sugar and oil consumption by 30% in three

years.

• It also aims to engage and enable citizens to improve their health and well-being

by making the right food choices.

Measures in place:

• FSSAI has put in place robust regulatory measures under three major pillars: Eat

Safe, Eat Health and Eat Sustainably for the programme.

• FSSAI has prescribed a limit for Total Polar Compounds (TPC) at 25% in cooking

oil to avoid the harmful effects of reused cooking oil.

https://www.insightsonindia.com/2019/12/05/eat-right-movement-campaign-2/

52. Consider the following statements regarding the features of Charter Act of 1813

1. It abolished the trade monopoly of the company in India and China.

2. It authorized the Local Governments in India to impose taxes on persons.

Which of the statements given above is/are correct?

(a) 1 only

(b) 2 only

(c) Both 1 and 2

(d) Neither 1 nor 2

Solution: B

Charter Act of 1813

The features of this Act were as follows:

• It abolished the trade monopoly of the company in India i.e., the Indian trade was

thrown open to all British merchants. However, it continued the monopoly of the

company over trade in tea and trade with China.

Page 53: SIMPLYFYING IAS EXAM PREPARATION - INSIGHTSIAS · 2020-04-13 · Insta 75 Days Revision Plan for UPSC Civil Services Prelims – 2020 This document is the compilation of 100 questions

INSTA 75 Days REVISION PLAN for Prelims 2020 - InstaTests

www.insightsonindia.com 51 Insights IAS

• It asserted the sovereignty of the British Crown over the Company’s territories in

India.

• It allowed the Christian missionaries to come to India for the purpose of

enlightening the people.

• It provided for the spread of western education among the inhabitants of the British

territories in India.

• It authorized the Local Governments in India to impose taxes on persons. They

could also punish the persons for not paying taxes.

53. Which of the following schedule deals with the provisions relating to the

administration of tribal areas?

(a) Fourth Schedule

(b) Eighth Schedule

(c) Sixth Schedule

(d) Seventh Schedule

Solution: C

Fourth Schedule

• Allocation of seats in the Rajya Sabha to the states and the union territories.

Fifth Schedule

• Provisions relating to the administration and control of scheduled areas and

scheduled tribes.

Sixth Schedule

• Provisions relating to the administration of tribal areas in the states of Assam,

Meghalaya, Tripura and Mizoram.

Seventh Schedule

• Division of powers between the Union and the States in terms of List I (Union

List), List II (State List) and List III (Concurrent List). Presently, the Union List

contains 98 subjects (originally 97), the State List contains 59 subjects (originally

66) and the Concurrent List contains 52 subjects (originally 47).

Eighth Schedule

• Languages recognized by the Constitution. Originally, it had 14 languages but

presently there are 22 languages.

Page 54: SIMPLYFYING IAS EXAM PREPARATION - INSIGHTSIAS · 2020-04-13 · Insta 75 Days Revision Plan for UPSC Civil Services Prelims – 2020 This document is the compilation of 100 questions

INSTA 75 Days REVISION PLAN for Prelims 2020 - InstaTests

www.insightsonindia.com 52 Insights IAS

Ninth Schedule

• Acts and Regulations (originally 13 but presently 282) of the state legislatures

dealing with land reforms and abolition of the zamindari system and of the

Parliament dealing with other matters. This schedule was added by the 1st

Amendment (1951) to protect the laws included in it from judicial scrutiny on the

ground of violation of fundamental rights. However, in 2007, the Supreme Court

ruled that the laws included in this schedule after April 24, 1973, are now open to

judicial review.

54. Consider the following statements regarding National Company Law Appellate

Tribunal (NCLAT)

1. It was constituted under the Companies Act, 2013

2. It is the Appellate Tribunal to hear and dispose of appeals against any

direction issued or decision made or order passed by the Competition

Commission of India (CCI).

Which of the statements given above is/are correct?

(a) 1 only

(b) 2 only

(c) Both 1 and 2

(d) Neither 1 nor 2

Solution: C

Government to set up National Company Law Appellate Tribunal Bench in Chennai.

About NCLAT:

• National Company Law Appellate Tribunal (NCLAT) was constituted under

Section 410 of the Companies Act, 2013.

Functions:

• It hears appeals against the orders of National Company Law Tribunal(s) (NCLT),

with effect from 1st June, 2016.

• It is the Appellate Tribunal for hearing appeals against the orders passed by

NCLT(s) under Section 61 of the Insolvency and Bankruptcy Code, 2016 (IBC).

• It is the Appellate Tribunal for hearing appeals against the orders passed by

Insolvency and Bankruptcy Board of India under Section 202 and Section 211 of

IBC.

Page 55: SIMPLYFYING IAS EXAM PREPARATION - INSIGHTSIAS · 2020-04-13 · Insta 75 Days Revision Plan for UPSC Civil Services Prelims – 2020 This document is the compilation of 100 questions

INSTA 75 Days REVISION PLAN for Prelims 2020 - InstaTests

www.insightsonindia.com 53 Insights IAS

• It is the Appellate Tribunal to hear and dispose of appeals against any direction

issued or decision made or order passed by the Competition Commission of India

(CCI).

Composition:

• The President of the Tribunal and the chairperson and Judicial Members of the

Appellate Tribunal shall be appointed after consultation with the Chief Justice of

India.

The Members of the Tribunal and the Technical Members of the Appellate Tribunal shall

be appointed on the recommendation of a Selection Committee consisting of:

1. Chief Justice of India or his nominee—Chairperson.

2. A senior Judge of the Supreme Court or a Chief Justice of High Court— Member.

3. Secretary in the Ministry of Corporate Affairs—Member.

4. Secretary in the Ministry of Law and Justice—Member.

5. Secretary in the Department of Financial Services in the Ministry of Finance—

Member.

https://www.insightsonindia.com/2019/12/03/national-company-law-appellate-

tribunal-nclat/

55. The phrase “assuring the dignity of the individual and the unity and integrity of the

Nation” is a part of

(a) Preamble

(b) Fundamental Rights

(c) Directive Principles of State Policy

(d) Fundamental Duties

Solution: A

The Preamble in its present form reads:

“We, THE PEOPLE OF INDIA, having solemnly resolved to constitute

India into a SOVEREIGN SOCIALIST SECULAR DEMOCRATIC REPUBLIC and

to secure to all its citizens:

JUSTICE, Social, Economic and Political;

LIBERTY of thought, expression, belief, faith and worship;

EQUALITY of status and of opportunity; and to promote among them all;

Page 56: SIMPLYFYING IAS EXAM PREPARATION - INSIGHTSIAS · 2020-04-13 · Insta 75 Days Revision Plan for UPSC Civil Services Prelims – 2020 This document is the compilation of 100 questions

INSTA 75 Days REVISION PLAN for Prelims 2020 - InstaTests

www.insightsonindia.com 54 Insights IAS

FRATERNITY assuring the dignity of the individual and the unity and integrity of the

Nation;

IN OUR CONSTITUENT ASSEMBLY this twenty-sixth day of November, 1949, do

HEREBY ADOPT, ENACT AND GIVE TO OURSELVES THIS

CONSTITUTION”.

56. Consider the following statements regarding Loss of Citizenship by Renunciation

1. It has been prescribed under Article 11 of the Constitution.

2. Any citizen of India of full age and capacity can make a declaration

renouncing his Indian citizenship.

3. If such a declaration is made during a war in which India is engaged, its

registration shall be withheld by the Central Government.

Which of the statements given above is/are correct?

(a) 1 and 2 only

(b) 2 and 3 only

(c) 1 and 3 only

(d) 1, 2 and 3

Solution: B

Loss of Citizenship

• The Citizenship Act (1955) prescribes three ways of losing citizenship whether

acquired under the Act or prior to it under the Constitution, viz, renunciation,

termination and deprivation:

By Renunciation

• Any citizen of India of full age and capacity can make a declaration renouncing his

Indian citizenship. Upon the registration of that declaration, that person ceases to

be a citizen of India. However, if such a declaration is made during a war in which

India is engaged, its registration shall be withheld by the Central Government.

• Further, when a person renounces his Indian citizenship, every minor child of that

person also loses Indian citizenship. However, when such a child attains the age of

eighteen, he may resume Indian citizenship.

By Termination

• When an Indian citizen voluntarily (consciously, knowingly and without duress,

undue influence or compulsion) acquires the citizenship of another country, his

Indian citizenship automatically terminates. This provision, however, does not

apply during a war in which India is engaged.

Page 57: SIMPLYFYING IAS EXAM PREPARATION - INSIGHTSIAS · 2020-04-13 · Insta 75 Days Revision Plan for UPSC Civil Services Prelims – 2020 This document is the compilation of 100 questions

INSTA 75 Days REVISION PLAN for Prelims 2020 - InstaTests

www.insightsonindia.com 55 Insights IAS

By Deprivation

It is a compulsory termination of Indian citizenship by the Central government, if:

(a) the citizen has obtained the citizenship by fraud:

(b) the citizen has shown disloyalty to the Constitution of India:

(c) the citizen has unlawfully traded or communicated with the enemy during

a war;

(d) the citizen has, within five years after registration or naturalization, been

imprisoned in any country for two years; and

(e) the citizen has been ordinarily resident out of India for seven years

Continuously.

57. Consider the following statements regarding Banking ombudsman scheme

1. It is an expeditious and inexpensive forum for bank customers for resolution

of complaints relating to certain services rendered by banks.

2. All Scheduled Commercial Banks, Regional Rural Banks and Scheduled

Primary Co-operative Banks are covered under the scheme.

3. The Banking Ombudsman is a senior official appointed by the Central

government to redress customer complaints against deficiency in certain

banking services.

Which of the statements given above is/are correct?

(a) 1 only

(b) 2 and 3 only

(c) 1 and 3 only

(d) 1 and 2 only

Solution: D

The Banking Ombudsman Scheme is an expeditious and inexpensive forum for bank

customers for resolution of complaints relating to certain services rendered by banks. The

Banking Ombudsman Scheme is introduced under Section 35 A of the Banking

Regulation Act, 1949 by RBI with effect from 1995. Presently the Banking Ombudsman

Scheme 2006 (As amended upto July 1, 2017) is in operation.

Who is a Banking Ombudsman?

• The Banking Ombudsman is a senior official appointed by the Reserve Bank of

India to redress customer complaints against deficiency in certain banking services

covered under the grounds of complaint specified under Clause 8 of the Banking

Ombudsman Scheme 2006 (As amended upto July 1, 2017).

Page 58: SIMPLYFYING IAS EXAM PREPARATION - INSIGHTSIAS · 2020-04-13 · Insta 75 Days Revision Plan for UPSC Civil Services Prelims – 2020 This document is the compilation of 100 questions

INSTA 75 Days REVISION PLAN for Prelims 2020 - InstaTests

www.insightsonindia.com 56 Insights IAS

How many Banking Ombudsmen have been appointed and where are they located?

• As on date, twenty Banking Ombudsmen have been appointed with their offices

located mostly in state capitals. The addresses and contact details of the Banking

Ombudsman offices have been provided under Annex I of the Scheme.

Which are the banks covered under the Banking Ombudsman Scheme, 2006?

• All Scheduled Commercial Banks, Regional Rural Banks and Scheduled Primary

Co-operative Banks are covered under the Scheme.

What are the grounds of complaints?

• The Banking Ombudsman can receive and consider any complaint relating to the

following deficiency in banking services:

https://m.rbi.org.in/Scripts/FAQView.aspx?Id=24

58. Consider the following statements regarding the Article 20 of the Indian

constitution:

1. It is available for only Indian citizen.

2. It grants protection against arbitrary and excessive punishment to legal

person like a company or a corporation.

Which of the statements given above is/are correct?

(a) 1 only

(b) 2 only

(c) Both 1 and 2

(d) Neither 1 nor 2

Solution: B

Protection in Respect of Conviction for Offences

Article 20 grants protection against arbitrary and excessive punishment to an accused

person, whether citizen or foreigner or legal person like a company or a corporation. It

contains three provisions in that direction:

(a) No ex-post-facto law: No person shall be (i) convicted of any offence except

for violation of a law in force at the time of the commission of the act, nor

(ii) subjected to a penalty greater than that prescribed by the law in force at

the time of the commission of the act.

(b) No double jeopardy: No person shall be prosecuted and punished for the

same offence more than once.

Page 59: SIMPLYFYING IAS EXAM PREPARATION - INSIGHTSIAS · 2020-04-13 · Insta 75 Days Revision Plan for UPSC Civil Services Prelims – 2020 This document is the compilation of 100 questions

INSTA 75 Days REVISION PLAN for Prelims 2020 - InstaTests

www.insightsonindia.com 57 Insights IAS

(c) No self-incrimination: No person accused of any offence shall be

compelled to be a witness against himself.

An ex-post-facto law is one that imposes penalties retrospectively (retroactively), that is,

upon acts already done or which increases the penalties for such acts. The enactment of

such a law is prohibited by the first provision of Article 20. However, this limitation is

imposed only on criminal laws and not on civil laws or tax laws. In other words, a civil

liability or a tax can be imposed retrospectively. Further, this provision prohibits only

conviction or sentence under an ex-post-facto criminal law and not the trial thereof.

Finally, the protection (immunity) under this provision cannot be claimed in case of

preventive detention or demanding security from a person.

The protection against double jeopardy is available only in proceedings before a court of

law or a judicial tribunal. In other words, it is not available in proceedings before

departmental or administrative authorities as they are not of judicial nature.

The protection against self-incrimination extends to both oral evidence and documentary

evidence. However, it does not extend to (i) compulsory production of material objects,

(ii) compulsion to give thumb impression, specimen signature, blood specimens, and (iii)

compulsory exhibition of the body. Further, it extends only to criminal proceedings and

not to civil proceedings or proceedings which are not of criminal nature.

59. Consider the following statements regarding Indian federation

1. The term ‘federation’ has nowhere been used in the Constitution.

2. The Indian federal system is based on the ‘American model’ of federation.

Which of the statements given above is/are correct?

(a) 1 only

(b) 2 only

(c) Both 1 and 2

(d) Neither 1 nor 2

Solution: A

The term ‘federation’ has nowhere been used in the Constitution. Instead, Article 1 of

the Constitution describes India as ‘Union of States’. According to Dr. B.R. Ambedkar,

the phrase ‘Union of States’ has been preferred to ‘Federation of States’ to indicate two

things:

(i) the Indian federation is not the result of an agreement among the states like the

American federation; and

(ii) the states have no right to secede from the federation. The federation is union

because it is indestructible.

Page 60: SIMPLYFYING IAS EXAM PREPARATION - INSIGHTSIAS · 2020-04-13 · Insta 75 Days Revision Plan for UPSC Civil Services Prelims – 2020 This document is the compilation of 100 questions

INSTA 75 Days REVISION PLAN for Prelims 2020 - InstaTests

www.insightsonindia.com 58 Insights IAS

The Indian federal system is based on the ‘Canadian model’ and not on the ‘American

model’. The ‘Canadian model’ differs fundamentally from the ‘American model’ in so far

as it establishes a very strong centre. The Indian federation resembles the Canadian

federation

(i) in its formation (i.e., by way of disintegration);

(ii) in its preference to the term ‘Union’ (the Canadian federation is also called a

‘Union’); and

(iii) in its centralizing tendency (i.e., vesting more powers in the centre vis-a-vis the

states).

60. Consider the following statements regarding Payment Banks

1. They cannot set up subsidiaries to undertake non-banking financial services

activities

2. They are not bounded by the reserve requirement rules of RBI

3. Non-Banking Finance Companies and corporate Banking Correspondents

can setup Payment Banks

Which of the statements given above is/are correct?

(a) 3 only

(b) 1 only

(c) 1 and 2 only

(d) 1 and 3 only

Solution: D

Payments Banks are a new set of banks licensed by the Reserve Bank of India to further

financial inclusion by enabling them to provide

(i) small savings/ current accounts below Rs. 1 lakh

(ii), distribution of mutual funds, insurance products on a non risk sharing basis and

(iii) payments / remittance services to migrant labour workforce, low income households,

small businesses, other unorganised sector entities and other users through high volume-

low value transactions in deposits and payments / remittance services using a secured

technology-driven environment including issuance of prepaid cards etc.

Payments Banks are differentiated or restricted banks. The Payments Bank cannot set up

subsidiaries to undertake non-banking financial services activities (hire purchase,

leasing etc.) nor can it undertake lending business.

Page 61: SIMPLYFYING IAS EXAM PREPARATION - INSIGHTSIAS · 2020-04-13 · Insta 75 Days Revision Plan for UPSC Civil Services Prelims – 2020 This document is the compilation of 100 questions

INSTA 75 Days REVISION PLAN for Prelims 2020 - InstaTests

www.insightsonindia.com 59 Insights IAS

It may choose to become a banking correspondent (BC) of another bank for credit and

other services which it cannot offer. Since liquidity is the most important aspect required

for such banks they will be bound by the reserve requirement rules of RBI (CRR, SLR

etc.).

Thus, apart from amounts maintained as Cash Reserve Ratio (CRR) with the Reserve

Bank on its outside demand and time liabilities, the payments banks will be required to

invest minimum 75 per cent of its “demand deposit balances” in Statutory Liquidity

Ratio (SLR) eligible Government securities/treasury bills with maturity up to one year

and hold maximum 25 per cent in current and time/fixed deposits with other scheduled

commercial banks for operational purposes and liquidity management.

The existing non-bank Pre-paid Payment Instruments Issuers (PPI issuers) authorised

under the Payment and Settlement Systems Act, 2007 (PSS Act) and other entities such

as Non-Banking Finance Companies (NBFCs), corporate Banking Correspondents (BCs),

mobile telephone companies, super-market chains, companies, real sector cooperatives

and public sector entities may apply to set up a Payments Bank.

Even banks can take equity stake in a Payments Bank to the extent permitted under

Section 19 (2) of the Banking Regulation Act, 1949.

http://arthapedia.in/index.php?title=Payment_Banks

61. On which of the following grounds can President declare a national emergency?

1. Grounds of war

2. Armed rebellion

3. External aggression

4. Internal disturbance

Select the correct answer using the code given below:

(a) 1, 2 and 3 only

(b) 2, 3 and 4 only

(c) 1, 3 and 4 only

(d) 1, 2, 3 and 4

Solution: A

Grounds of Declaration

• Under Article 352, the President can declare a national emergency when the

security of India or a part of it is threatened by war or external aggression or armed

rebellion. It may be noted that the president can declare a national emergency

even before the actual occurrence of war or external aggression or armed

rebellion, if he is satisfied that there is an imminent danger.

Page 62: SIMPLYFYING IAS EXAM PREPARATION - INSIGHTSIAS · 2020-04-13 · Insta 75 Days Revision Plan for UPSC Civil Services Prelims – 2020 This document is the compilation of 100 questions

INSTA 75 Days REVISION PLAN for Prelims 2020 - InstaTests

www.insightsonindia.com 60 Insights IAS

• The President can also issue different proclamations on grounds of war, external

aggression, armed rebellion, or imminent danger thereof, whether or not there is a

proclamation already issued by him and such proclamation is in operation. This

provision was added by the 38th Amendment Act of 1975. When a national

emergency is declared on the ground of ‘war’ or ‘external aggression’, it is known

as ‘External Emergency’. On the other hand, when it is declared on the ground of

‘armed rebellion’, it is known as ‘Internal Emergency’.

• A proclamation of national emergency may be applicable to the entire country or

only a part of it. The 42nd Amendment Act of 1976 enabled the president to limit

the operation of a National Emergency to a specified part of India.

• Originally, the Constitution mentioned ‘internal disturbance’ as the third ground

for the proclamation of a National Emergency, but the expression was too vague

and had a wider connotation.

• Hence, the 44th Amendment Act of 1978 substituted the words ‘armed rebellion’

for ‘internal disturbance’. Thus, it is no longer possible to declare a National

Emergency on the ground of ‘internal disturbance’ as was done in 1975 by the

Congress government headed by Indira Gandhi.

62. Consider the following statements regarding Committee of Privileges

1. The functions of this committee are semi-judicial in nature.

2. The committee has 25 members, 15 members from Lok Sabha and 10

members from Rajya Sabha committee.

Which of the statements given above is/are correct?

(a) 1 only

(b) 2 only

(c) Both 1 and 2

(d) Neither 1 nor 2

Solution: A

Committee of Privileges

• The functions of this committee are semi-judicial in nature. It examines the cases

of breach of privileges of the House and its members and recommends

appropriate action.

• There are separate committees for Lok sabha and Rajya Sabha

• The Lok Sabha committee has 15 members, while the Rajya Sabha committee

has 10 members.

Page 63: SIMPLYFYING IAS EXAM PREPARATION - INSIGHTSIAS · 2020-04-13 · Insta 75 Days Revision Plan for UPSC Civil Services Prelims – 2020 This document is the compilation of 100 questions

INSTA 75 Days REVISION PLAN for Prelims 2020 - InstaTests

www.insightsonindia.com 61 Insights IAS

63. Consider the following statements regarding PM-Kisan Scheme

1. It is a centrally sponsored scheme with 50% contribution from centre.

2. All Institutional Land holders are excluded from the scheme

3. The Common Service Centres (CSCs) have been authorized to do

registration of the farmers for the scheme upon payment of fees.

Which of the statements given above is/are correct?

(a) 2 and 3 only

(b) 1 only

(c) 3 only

(d) 1, 2 and 3

Solution: A

PM- KISAN Scheme

• Pradhan Mantri Kisan Samman Nidhi (PM-KISAN) is a Central Sector scheme

with 100% funding from Government of India.

• The Scheme is effective from 1.12.2018.

• Under the Scheme an income support of Rs.6000/- per year is provided to all

farmer families across the country in three equal installments of Rs.2000/- each

every four months.

• Definition of family for the Scheme is husband, wife and minor children.

• The entire responsibility of identification of beneficiary farmer families rests with

the State / UT Governments.

• The fund is directly transferred to the bank accounts of the beneficiaries.

• Farmers covered under the Exclusion Criteria of the Operational Guidelines are

not eligible for the benefit of the Scheme.

• For enrollment, the farmer is required to approach the local patwari / revenue

officer / Nodal Officer (PM-Kisan) nominated by the State Government.

• The Common Service Centres (CSCs) have also been authorized to do registration

of the farmers for the Scheme upon payment of fees.

• All Institutional Land holders are excluded from the scheme

https://pmkisan.gov.in/

64. Consider the following statements regarding composition of judges

1. The President determines the strength of a supreme court from time to time

depending upon its workload.

2. At present, the Supreme Court consists of thirty-four judges (one chief

justice and thirty-three other judges).

Page 64: SIMPLYFYING IAS EXAM PREPARATION - INSIGHTSIAS · 2020-04-13 · Insta 75 Days Revision Plan for UPSC Civil Services Prelims – 2020 This document is the compilation of 100 questions

INSTA 75 Days REVISION PLAN for Prelims 2020 - InstaTests

www.insightsonindia.com 62 Insights IAS

Which of the statements given above is/are correct?

(a) 1 only

(b) 2 only

(c) Both 1 and 2

(d) Neither 1 nor 2

Solution: B

Article 124: Establishment and Constitution of Supreme Court

(1) There shall be a Supreme Court of India consisting of a Chief Justice of India and, until

Parliament by law prescribes a larger number, of not more than seven other Judges.

COMPOSITION AND APPOINTMENT

• At present, the Supreme Court consists of thirty-four judges (one chief justice and

thirty-three other judges). In 2019, the centre notified an increase in the number

of Supreme Court judges from thirty-one to thirty-four, including the Chief

Justice of India. This followed the enactment of the Supreme Court (Number of

Judges) Amendment Act, 2019. Originally, the strength of the Supreme Court was

fixed at eight (one chief justice and seven other judges). The Parliament has

increased this number of other judges progressively to ten in 1956, to thirteen in

1960, to seventeen in 1977, to twenty-five in 1986, to thirty in 2008 and to thirty-

three in 2019.

Article 216 in The Constitution Of India 1949

• 216 Constitution of High Courts Every High Court shall consist of a Chief Justice

and such other Judges as the President may from time to time deem it necessary to

appoint

• Every high court (whether exclusive or common) consists of a chief justice and such

other judges as the president may from time to time deem necessary to appoint.

Thus, the Constitution does not specify the strength of a high court and leaves it to

the discretion of the president. Accordingly, the President determines the strength

of a high court from time to time depending upon its workload.

Page 65: SIMPLYFYING IAS EXAM PREPARATION - INSIGHTSIAS · 2020-04-13 · Insta 75 Days Revision Plan for UPSC Civil Services Prelims – 2020 This document is the compilation of 100 questions

INSTA 75 Days REVISION PLAN for Prelims 2020 - InstaTests

www.insightsonindia.com 63 Insights IAS

65. Consider the following statements regarding District Planning Committee

1. Every state shall constitute at the district level, a district planning committee

2. The representation of these members in the committee should be in

proportion to the ratio between the rural and urban populations in the

district.

3. Four-fifths of the members of a district planning committee should be

elected by the elected members of the district panchayat and municipalities

in the district from amongst themselves.

Which of the statements given above is/are correct?

(a) 1 and 2 only

(b) 2 and 3 only

(c) 1 and 3 only

(d) 1, 2 and 3

Solution: D

District Planning Committee

Every state shall constitute at the district level, a district planning committee to consolidate

the plans prepared by panchayats and municipalities in the district, and to prepare a draft

development plan for the district as a whole. The state legislature may make provisions

with respect to the following:

The composition of such committees;

• The manner of election of members of such committees;

• The functions of such committees in relation to district planning; and

• The manner of the election of the chairpersons of such committees.

• The act lays down that four-fifths of the members of a district planning committee

should be elected by the elected members of the district panchayat and

municipalities in the district from amongst themselves. The representation of these

members in the committee should be in proportion to the ratio between the rural

and urban populations in the district.

• The chairperson of such committee shall forward the development plan to the state

government.

In preparing the draft development plan, a district planning committee shall

(a) Have regard to–

(i) matters of common interest between the Panchayats and Municipalities

including spatial planning, sharing of water other physical and natural

resources, the integrated development of infrastructure and

environmental conservation;

Page 66: SIMPLYFYING IAS EXAM PREPARATION - INSIGHTSIAS · 2020-04-13 · Insta 75 Days Revision Plan for UPSC Civil Services Prelims – 2020 This document is the compilation of 100 questions

INSTA 75 Days REVISION PLAN for Prelims 2020 - InstaTests

www.insightsonindia.com 64 Insights IAS

(ii) the extent and type of available resources whether financial otherwise;

and

(b) Consult such institutions and organizations as the Governor may specify.

66. International Intellectual Property (IP) Index, sometime seen in the news, is

published by

(a) World intellectual property organisation

(b) World Trade Organisation

(c) World Economic Forum

(d) US Chambers of Commerce

Solution: D

India’s slipped to 40th position on the International Intellectual Property (IP) Index, which

analyses the IP climate in 53 global economies, this year, according to a report of US

Chamber of Commerce’s Global Innovation Policy Center. Last year India was ranked

at 36th position out of 50 countries.

The US, the UK, Sweden, France and Germany remained the top five economies on the

intellectual property index in 2019 retaining their spots from the last year.

“Since the release of the 2016 National IPR Policy, the Government of India has made a

focused effort to support investments in innovation and creativity through increasingly

robust IP protection and enforcement,” said Patrick Kilbride, Senior Vice President for the

Global Innovation Policy Center.

67. Consider the following statements regarding State Human Rights Commission

1. It can inquire into violation of human rights only in respect of subjects

mentioned in the State List (List-II) and the Concurrent List (List-III).

2. The chairperson should be a retired Chief Justice or a Judge of a Supreme

Court.

Which of the statements given above is/are correct?

(a) 1 only

(b) 2 only

(c) Both 1 and 2

(d) Neither 1 nor 2

Page 67: SIMPLYFYING IAS EXAM PREPARATION - INSIGHTSIAS · 2020-04-13 · Insta 75 Days Revision Plan for UPSC Civil Services Prelims – 2020 This document is the compilation of 100 questions

INSTA 75 Days REVISION PLAN for Prelims 2020 - InstaTests

www.insightsonindia.com 65 Insights IAS

Solution: A

The Protection of Human Rights Act of 1993 provides for the creation of not only the

National Human Rights Commission but also a State Human Rights Commission at the

state level. Accordingly, twenty-six states have constituted the State Human Rights

Commissions through Official Gazette Notifications.

• A State Human Rights Commission can inquire into violation of human rights

only in respect of subjects mentioned in the State List (List-II) and the

Concurrent List (List-III) of the Seventh Schedule of the Constitution. However,

if any such case is already being inquired into by the National Human Rights

Commission or any other Statutory Commission, then the State Human Rights

Commission does not inquire into that case.

• The State Human Rights Commission is a multi-member body consisting of a

chairperson and two members. The chairperson should be a retired Chief Justice

or a Judge of a High Court and members should be a serving or retired judge of a

High Court or a District Judge in the state with a minimum of seven years’

experience as District Judge and a person having knowledge or practical experience

with respect to human rights.

68. Consider the following statements regarding GOBAR-DHAN scheme

1. Ministry of Drinking Water & Sanitation has launched the GOBAR

(Galvanizing Organic Bio-Agro Resources) – DHAN scheme.

2. The scheme aims to positively impact village cleanliness and generate

wealth and energy from cattle and organic waste.

Which of the statements given above is/are correct?

(a) 1 only

(b) 2 only

(c) Both 1 and 2

(d) Neither 1 nor 2

Solution: C

GOBAR-DHAN scheme

• Ministry of Drinking Water & Sanitation has launched the GOBAR

(Galvanizing Organic Bio-Agro Resources) – DHAN scheme. The scheme is

being implemented as part of the Swachh Bharat Mission (Gramin).

• The Swachh Bharat Mission (Gramin) comprises two main components for

creating clean villages – creating open defecation free (ODF) villages and

Page 68: SIMPLYFYING IAS EXAM PREPARATION - INSIGHTSIAS · 2020-04-13 · Insta 75 Days Revision Plan for UPSC Civil Services Prelims – 2020 This document is the compilation of 100 questions

INSTA 75 Days REVISION PLAN for Prelims 2020 - InstaTests

www.insightsonindia.com 66 Insights IAS

managing solid and liquid waste in villages. With over 3.5 lakh villages, 374

districts and 16 States/UTsof the country being declared ODF, the stage is set for

ODF-plus activities, including measures to enhance Solid and Liquid Waste

Management (SLWM). The GOBAR-DHAN scheme, with its focus on keeping

villages clean, increasing the income of rural households, and generation of energy

from cattle waste, is an important element of this ODF-plus strategy.

Aim

• The scheme aims to positively impact village cleanliness and generate wealth and

energy from cattle and organic waste. The scheme also aims at creating new rural

livelihood opportunities and enhancing income for farmers and other rural people.

69. Consider the following statements regarding Nilavembu kudineer

1. It is a Siddha medicine

2. It provides protection against chikungunya virus.

Which of the statements given above is/are correct?

(a) 1 only

(b) 2 only

(c) Both 1 and 2

(d) Neither 1 no 2

Solution: C

Nilavembu kudineer is a siddha medicine which kills dengue virus, protects from

Chikungunya. The siddha drug showed significant antiviral activity,

immunomodulation.

(Tamil Nadu government had distributed nilavembu kudineer concoction to treat people

infected with dengue during the outbreak in late 2017)

https://www.thehindu.com/sci-tech/science/nilavembu-kudineer-kills-dengue-virus-

protects-from-chikungunya/article26161732.ece

70. Which of the following countries is/are members of OPEC?

1. Iran

2. Saudi Arabia

3. Russia

4. USA

Select the correct answer using the code given below:

Page 69: SIMPLYFYING IAS EXAM PREPARATION - INSIGHTSIAS · 2020-04-13 · Insta 75 Days Revision Plan for UPSC Civil Services Prelims – 2020 This document is the compilation of 100 questions

INSTA 75 Days REVISION PLAN for Prelims 2020 - InstaTests

www.insightsonindia.com 67 Insights IAS

(a) 2 only

(b) 1 and 2 only

(c) 1, 2 and 3 only

(d) 1, 2, 3 and 4

Solution: B

Organization of the Petroleum Exporting Countries (OPEC):

• The Organization of the Petroleum Exporting Countries (OPEC) was founded in

Baghdad, Iraq, with the signing of an agreement in September 1960 by five

countries namely Islamic Republic of Iran, Iraq, Kuwait, Saudi Arabia and

Venezuela. They were to become the Founder Members of the Organization.

• It is a permanent, intergovernmental organization.

• Currently, the Organization has a total of 13 Member Countries.

• The current OPEC members are the following: Algeria, Angola, Equatorial

Guinea, Gabon, Iran, Iraq, Kuwait, Libya, Nigeria, the Republic of the Congo,

Saudi Arabia (the de facto leader), the United Arab Emirates and Venezuela.

Ecuador, Indonesia and Qatar are members.

• OPEC’s objective is to co-ordinate and unify petroleum policies among Member

Countries, in order to secure fair and stable prices for petroleum producers; an

efficient, economic and regular supply of petroleum to consuming nations; and a

fair return on capital to those investing in the industry.

• It is headquartered in Vienna, Austria.

• OPEC membership is open to any country that is a substantial exporter of oil and

which shares the ideals of the organization.

What is the Opec+?

• Opec+ refers to the alliance of crude producers, who have been undertaking

corrections in supply in the oil markets since 2017.

• OPEC plus countries include Azerbaijan, Bahrain, Brunei, Kazakhstan,

Malaysia, Mexico, Oman, Russia, South Sudan and Sudan.

What are their goals?

• The Opec and non-Opec producers first formed the alliance at a historic meeting

in Algiers in 2016.

• The aim was to undertake production restrictions to help resuscitate a flailing

market.

Page 70: SIMPLYFYING IAS EXAM PREPARATION - INSIGHTSIAS · 2020-04-13 · Insta 75 Days Revision Plan for UPSC Civil Services Prelims – 2020 This document is the compilation of 100 questions

INSTA 75 Days REVISION PLAN for Prelims 2020 - InstaTests

www.insightsonindia.com 68 Insights IAS

71. The 6x6x6 strategy, sometimes seen in news is related to the tackling of:

(a) Anemia

(b) Malaria

(c) Covid -19

(d) Leprosy

Solution: A

The anemia control programme review indicated challenges in IFA supply chain

management, demand generation and monitoring. Considering, the slow progress i.e. less

than 1% per annum in reduction of anaemia from 2005 to 2015, the Government of India

has launched the Anemia Mukt Bharat (AMB) strategy under the Prime Minister’s

Overarching Scheme for Holistic Nourishment (POSHAN) Abhiyaan and the targets

has been set to reduce anaemia by 3% per year.

The 6x6x6 strategy under AMB implies six age groups, six interventions and six

institutional mechanisms. The strategy focuses on ensuring supply chain, demand

generation and strong monitoring using the dashboard for addressing anemia, both due to

nutritional and non-nutritional causes.

(A) The six population groups under AMB strategy are:

1) Children (6-59 months)

2) Children (5-9 years)

3) Adolescents girls and boys (10-19 years)

4) Pregnant women

5) Lactating women

6) Women of Reproductive Age (WRA) group (15-49 years)

(B) The six interventions are:

1) Prophylactic Iron and Folic Acid Supplementation

2) Deworming

3) Intensified year-round Behaviour Change Communication (BCC) Campaign and

delayed cord clamping

4) Testing of anaemia using digital methods and point of care treatment,

5) Mandatory provision of Iron and Folic Acid fortified foods in Government funded

health programmes

6) Addressing non-nutritional causes of anaemia in endemic pockets with special

focus on malaria, hemoglobinopathies and fluorosis and the six institutional

mechanisms.

Page 71: SIMPLYFYING IAS EXAM PREPARATION - INSIGHTSIAS · 2020-04-13 · Insta 75 Days Revision Plan for UPSC Civil Services Prelims – 2020 This document is the compilation of 100 questions

INSTA 75 Days REVISION PLAN for Prelims 2020 - InstaTests

www.insightsonindia.com 69 Insights IAS

(C) The six institutional mechanisms are:

1) Inter-ministerial coordination

2) National AnemiaMukt Bharat Unit

3) National Centre of Excellence and Advanced research on Anemia Control

4) Convergence with other ministries

5) Strengthening supply chain and logistics

6) Anemia Mukt Bharat Dashboard and Digital Portal- one-stop shop for Anemia.

This comprehensive strategy is expected to yield positive results in Anemia control.

72. Consider the following statements regarding ‘UMMID’ initiative

1. It is an initiative of Ministry of Health and Family Welfare.

2. It is an initiative to tackle inherited genetic diseases of new born babies.

Which of the statements given above is/are correct?

(a) 1 only

(b) 2 only

(c) Both 1 and 2

(d) Neither 1 nor 2

Solution: B

‘UMMID’ initiative

• Ministry of Science & Technology launches ‘UMMID’ initiative to tackle

inherited genetic diseases of new born babies.

UMMID (Unique Methods of Management and treatment of Inherited Disorders)

initiative:

• Department of Biotechnology has started the UMMID Initiative which is

designed on the concept of ‘Prevention is better than Cure’.

• UMMID aims to create awareness about genetic disorders amongst clinicians

and establish molecular diagnostics in hospitals so that the fruits of developments

in medical genetics reach the patients in India.

UMMID initiative aims to:

• Establish NIDAN (National Inherited Diseases Administration) Kendras to

provide counselling, prenatal testing and diagnosis, management, and

multidisciplinary care in Government Hospitals wherein the influx of patients is

more.

• Produce skilled clinicians in Human Genetics.

Page 72: SIMPLYFYING IAS EXAM PREPARATION - INSIGHTSIAS · 2020-04-13 · Insta 75 Days Revision Plan for UPSC Civil Services Prelims – 2020 This document is the compilation of 100 questions

INSTA 75 Days REVISION PLAN for Prelims 2020 - InstaTests

www.insightsonindia.com 70 Insights IAS

• Undertake screening of pregnant women and new born babies for inherited genetic

diseases in hospitals at aspirational districts.

73. Aureofungin, Kasugamycin and Validamycin, sometimes seen in news, are:

(a) Pesticides

(b) New butterfly species found in Western Ghats

(c) Bacterial diseases prevalent in wild animals

(d) New elements added to periodic table.

Solution: A

Aureofungin, Kasugamycin, Validamycin and Streptomycin+ Tetracycline

combination are antibiotics that are registered under the Insecticide Act 1968 for use as

pesticides to combat certain fungal and bacterial diseases in plants.

Pesticides are toxic substances but they do not pose any adverse effect on human beings,

animals and the environment if they are used as per the label and leaflet approved by the

Central Insecticide Board and Registration Committee.

Extra Learning:

Central Insecticide Board and Registration Committee:

• The CIBRC is the country’s apex body that approves the use of pesticides.

The use of pesticides and insecticides in India are regulated by the Following two:

1) Insecticides Act, 1968

2) Insecticides Rules, 1971

In the Act and the Rules framed thereunder, there is compulsory registration of the

pesticides at the Central level and licence for their manufacture, formulation and sale are

dealt with at the State level.

The CIBRC functions under the Department of Agriculture, Ministry of Agriculture and

Farmers’ Welfare.

Page 73: SIMPLYFYING IAS EXAM PREPARATION - INSIGHTSIAS · 2020-04-13 · Insta 75 Days Revision Plan for UPSC Civil Services Prelims – 2020 This document is the compilation of 100 questions

INSTA 75 Days REVISION PLAN for Prelims 2020 - InstaTests

www.insightsonindia.com 71 Insights IAS

74. Consider the following statements regarding The United Nations Conference on

Trade and Development (UNCTAD)

1. UNCTAD is a permanent intergovernmental body established by the

United Nations General Assembly and forms a part of the United Nations

Development Group.

2. It reports to the UN General Assembly and the Economic and Social

Council.

Which of the statements given above is/are correct?

(a) 1 only

(b) 2 only

(c) Both 1 and 2

(d) Neither 1 nor 2

Solution: C

UNCTAD in the UN system

• UNCTAD is a permanent intergovernmental body established by the United

Nations General Assembly in 1964. Our headquarters are located in Geneva,

Switzerland, and have offices in New York and Addis Ababa.

• UNCTAD is part of the UN Secretariat. We report to the UN General Assembly

and the Economic and Social Council but have our own membership, leadership,

and budget. We are also part of the United Nations Development Group.

• Globalization, including a phenomenal expansion of trade, has helped lift millions

out of poverty. But not nearly enough people have benefited. And tremendous

challenges remain.

• We support developing countries to access the benefits of a globalized economy

more fairly and effectively. And we help equip them to deal with the potential

drawbacks of greater economic integration. To do this, we provide analysis,

facilitate consensus-building, and offer technical assistance. This helps them to use

trade, investment, finance, and technology as vehicles for inclusive and sustainable

development.

75. Consider the following statements regarding the Purana Qila:

1. The massive gateway and walls of the Qila were built by Mughal ruler

Jahangir.

2. The PGW (Painted Gray Ware Pottery) was recovered from the site.

3. It is located on the banks of river Betwa.

Which of the statements given above is/are correct?

(a) 2 only

Page 74: SIMPLYFYING IAS EXAM PREPARATION - INSIGHTSIAS · 2020-04-13 · Insta 75 Days Revision Plan for UPSC Civil Services Prelims – 2020 This document is the compilation of 100 questions

INSTA 75 Days REVISION PLAN for Prelims 2020 - InstaTests

www.insightsonindia.com 72 Insights IAS

(b) 1 only

(c) 1, 2 and 3

(d) None

Solution: A

The Government/Archaeological Survey of India (ASI) will set up a museum at Purana

Quila to house objects excavated from across India which have been lying in the Central

Antiquities Collection (CAC).

• Built on the site of the most ancient of the numerous cities of Delhi, Indraprastha,

Purana Quila is roughly rectangular in shape having a circuit of nearly two

kilometers.

• The thick ramparts crowned by merlons have three gateways provided with

bastions on either side. It was surrounded by a wide moat, connected to river

Yamuna, which used to flow on the east of the fort. The northern gate way, called

the Talaqui darwaza or the forbidden gateway, combines the typically Islamic

pointed arch with Hindu Chhatris and brackets; whereas the southern gateway

called the Humayun Darwaza also had a similar plan.

• The massive gateway and walls of Purana Quila were built by Humayun and the

foundation laid for the new capital, Dinpanah.

• The work was carried forward by Sher Shah Suri,who displaced Humayun, Purana

Quila is the venue for the spectacular sound and light show held every evening.

• Excavations have revealed that the Purana Quila stands at the site of Indraprastha,

the capital of the Pandavas. Excavations near one of the walls of the fort show that

the site had been occupied since 1000 B.C.

• The PGW (Painted Gray Ware Pottery) recovered from the site date back to

around 1000 B.C.

http://delhitourism.gov.in/delhitourism/tourist_place/purana_quila.jsp

• The Central Antiquity Collection Section is a centre for housing the collection of

antiquities explored and excavated by the Archaeological Survey of India.

• The CAC is under the ASI and the artefacts stored here are accessible only to

researchers.

Page 75: SIMPLYFYING IAS EXAM PREPARATION - INSIGHTSIAS · 2020-04-13 · Insta 75 Days Revision Plan for UPSC Civil Services Prelims – 2020 This document is the compilation of 100 questions

INSTA 75 Days REVISION PLAN for Prelims 2020 - InstaTests

www.insightsonindia.com 73 Insights IAS

DAY – 24 (InstaTest-24)

76. Consider the following statements regarding Gangetic river dolphins

1. The Gangetic river dolphins can only live in freshwater

2. They are found in parts of the Ganges-Meghna-Brahmaputra and

Karnaphuli-Sangu river systems

3. They have been included in Schedule I of the Indian Wild Life (Protection)

Act 1972

Which of the statements given above is/are correct?

(a) 1 only

(b) 1 and 2 only

(c) 2 and 3 only

(d) 1, 2 and 3

Solution: D

The government is planning to launch a programme called “Project Dolphin”, along the

lines of “Project Tiger” to enhance the population of these dolphins.

Protection status:

• Dolphins have been included in Schedule I of the Indian Wild Life (Protection)

Act 1972, in Appendix I of the Convention on International Trade in Endangered

Species (CITES), in Appendix II of the Convention on Migratory Species (CMS)

and categorised as ‘Endangered’ on the International Union for the Conservation

of Nature’s (IUCN) Red List.

Gangetic river dolphins:

• The Gangetic river dolphins can only live in freshwater, are blind and catch their

prey in a unique manner, using ultrasonic sound waves.

• They are distributed across seven states in India: Assam, Uttar Pradesh, Madhya

Pradesh, Rajasthan, Bihar, Jharkhand and West Bengal.

• The Ganges river dolphin is found in parts of the Ganges-Meghna-Brahmaputra

and Karnaphuli-Sangu river systems in India, Nepal, and Bangladesh.

https://www.insightsonindia.com/2019/12/17/project-dolphin/

Page 76: SIMPLYFYING IAS EXAM PREPARATION - INSIGHTSIAS · 2020-04-13 · Insta 75 Days Revision Plan for UPSC Civil Services Prelims – 2020 This document is the compilation of 100 questions

INSTA 75 Days REVISION PLAN for Prelims 2020 - InstaTests

www.insightsonindia.com 74 Insights IAS

77. Consider the following statements regarding taxation during mahajanapadas

1. Taxes on crops were the most important and it was fixed at 1/6th of the

produce.

2. There were taxes on crafts persons and it was in the form of labour.

3. Hunters and gatherers were not taxed.

Which of the statements given above is/are correct?

(a) 1 and 2 only

(b) 2 and 3 only

(c) 1 and 3 only

(d) 1, 2 and 3

Page 77: SIMPLYFYING IAS EXAM PREPARATION - INSIGHTSIAS · 2020-04-13 · Insta 75 Days Revision Plan for UPSC Civil Services Prelims – 2020 This document is the compilation of 100 questions

INSTA 75 Days REVISION PLAN for Prelims 2020 - InstaTests

www.insightsonindia.com 75 Insights IAS

Solution: A

The Mahajanapadas were sixteen kingdoms or oligarchic republics that existed in ancient

India from the sixth to fourth centuries BCE.

Taxes

As the rulers of the mahajanapadas were

(a) building huge forts

(b) maintaining big armies, they needed more resources.

• And they needed officials to collect these. So, instead of depending on occasional

gifts brought by people, as in the case of the raja of the janapadas, they started

collecting regular taxes.

• Taxes on crops were the most important. This was because most people were

farmers. Usually, the tax was fixed at 1/6th of what was produced. This was known

as bhaga or a share.

• There were taxes on crafts persons as well. These could have been in the form of

labour. For example, a weaver or a smith may have had to work for a day every

month for the king.

• Herders were also expected to pay taxes in the form of animals and animal produce.

• There were also taxes on goods that were bought and sold, through trade.

• And hunters and gatherers also had to provide forest produce to the raja.

78. Consider the following statements regarding the Upanishads

1. These were part of the later Vedic texts

2. There were no women Upanishadic thinkers.

Which of the statements given above is/are correct?

(a) 1 only

(b) 2 only

(c) Both 1 and 2

(d) Neither 1 nor 2

Solution: A

Aaranyak and Upanishad are last part of Brahman Granth which ponders over

philosophical questions. These throw light on religious and culture life of early Hindus.

Many of their ideas were recorded in the Upanishads. These were part of the later Vedic

texts. Upanishad literally means ‘approaching and sitting near’ and the texts contain

Page 78: SIMPLYFYING IAS EXAM PREPARATION - INSIGHTSIAS · 2020-04-13 · Insta 75 Days Revision Plan for UPSC Civil Services Prelims – 2020 This document is the compilation of 100 questions

INSTA 75 Days REVISION PLAN for Prelims 2020 - InstaTests

www.insightsonindia.com 76 Insights IAS

conversations between teachers and students. Often, ideas were presented through simple

dialogues.

Most Upanishadic thinkers were men, especially brahmins and rajas. Occasionally, there

is mention of women thinkers, such as Gargi, who was famous for her learning, and

participated in debates held in royal courts. Poor people rarely took part in these

discussions. One famous exception was Satyakama Jabala, who was named after his

mother, the slave woman Jabali. He had a deep desire to learn about reality, was accepted

as a student by a brahmin teacher named Gautama, and became one of the best-known

thinkers of the time.

79. Consider the following statements regarding National Ganga Council (NGC)

1. It was formed under Water (Prevention & Control of Pollution) Act, 1974.

2. The Prime Minister is the ex-officio Chairperson for the NGC.

Which of the statements given above is/are correct?

(a) 1 only

(b) 2 only

(c) Both 1 and 2

(d) Neither 1 nor 2

Solution: B

Review meeting of the National Ganga Council was recently held.

About Namami Gange Programme:

• It is an umbrella programme which integrates previous and currently ongoing

initiatives by enhancing efficiency, extracting synergies and supplementing them

with more comprehensive & better coordinated interventions.

• Implemented by the National Mission for Clean Ganga (NMCG), and its state

counterparts—State Programme Management Groups.

National Ganga Council (NGC):

• Created in October 2016 under the River Ganga (Rejuvenation, Protection and

Management) Authorities Order, 2016, dissolving the National Ganga River Basin

Authority.

• Headed by the Prime Minister.

• It replaced the National Ganga River Basin Authority (NGRBA).

• The National Ganga Council is formed under the Environment (Protection) Act

(EPA),1986.

Page 79: SIMPLYFYING IAS EXAM PREPARATION - INSIGHTSIAS · 2020-04-13 · Insta 75 Days Revision Plan for UPSC Civil Services Prelims – 2020 This document is the compilation of 100 questions

INSTA 75 Days REVISION PLAN for Prelims 2020 - InstaTests

www.insightsonindia.com 77 Insights IAS

• NGC would have on board the chief ministers of five Ganga basin states—

Uttarakhand, Uttar Pradesh (UP), Bihar, Jharkhand and West Bengal—besides

several Union ministers and it was supposed to meet once every year.

https://www.insightsonindia.com/2019/12/16/namami-gange-2/

80. Consider the following statements regarding Arthasastra

1. This book was written in Sanskrit language by Kautilya.

2. The manuscript of Arthasastra was first discovered by R. Shama Sastri.

Which of the statements given above is/are correct?

(a) 1 only

(b) 2 only

(c) Both 1 and 2

(d) Neither 1 nor 2

Solution: C

Kautilya’s Arthasastra

• This book in Sanskrit was written by Kautilya, a contemporary of Chandragupta

Maurya. Kautilya was also called ‘Indian Machiavelli’. The manuscript of

Arthasastra was first discovered by R. Shama Sastri in 1904.

• The Arthasastra contains 15 books and 180 chapters but it can be divided into three

parts: the first deals with the king and his council and the departments of

government; the second with civil and criminal law; and the third with diplomacy

and war. It is the most important literary source for the history of the Mauryas.

81. Consider the following statements regarding Agricultural technologies during

Harappan periods

1. Terracotta models of the plough have been found at sites in Cholistan and

at Banawali.

2. Evidence of a ploughed field was found at Kalibangan.

3. Traces of canals have been found at Shortughai in Afghanistan

Which of the statements given above is/are correct?

(a) 1 and 2 only

(b) 2 and 3 only

(c) 1 and 3 only

Page 80: SIMPLYFYING IAS EXAM PREPARATION - INSIGHTSIAS · 2020-04-13 · Insta 75 Days Revision Plan for UPSC Civil Services Prelims – 2020 This document is the compilation of 100 questions

INSTA 75 Days REVISION PLAN for Prelims 2020 - InstaTests

www.insightsonindia.com 78 Insights IAS

(d) 1, 2 and 3

Solution: D

Agricultural technologies

• While the prevalence of agriculture is indicated by finds of grain, it is more difficult

to reconstruct actual agricultural practices. Were seeds broadcast (scattered) on

ploughed lands? Representations on seals and terracotta sculpture indicate that the

bull was known, and archaeologists extrapolate from this that oxen were used for

ploughing. Moreover, terracotta models of the plough have been found at sites in

Cholistan and at Banawali (Haryana).

• Archaeologists have also found evidence of a ploughed field at Kalibangan

(Rajasthan), associated with Early Harappan levels. The field had two sets of

furrows at right angles to each other, suggesting that two different crops were grown

together.

• Archaeologists have also tried to identify the tools used for harvesting. Did the

Harappans use stone blades set in wooden handles or did they use metal tools?

• Most Harappan sites are located in semi-arid lands, where irrigation was probably

required for agriculture. Traces of canals have been found at the Harappan site of

Shortughai in Afghanistan, but not in Punjab or Sind.

82. Consider the following statements regarding Adaptation fund

1. It was set up under the Kyoto Protocol

2. It finances projects and programmes that help vulnerable communities in

developing countries adapt to climate change.

3. The Fund is supervised and managed by United Nations Environment

Program (UNEP).

Which of the statements given above is/are correct?

(a) 2 only

(b) 2 and 3 only

(c) 1 and 2 only

(d) 1, 2 and 3

Solution: C

Page 81: SIMPLYFYING IAS EXAM PREPARATION - INSIGHTSIAS · 2020-04-13 · Insta 75 Days Revision Plan for UPSC Civil Services Prelims – 2020 This document is the compilation of 100 questions

INSTA 75 Days REVISION PLAN for Prelims 2020 - InstaTests

www.insightsonindia.com 79 Insights IAS

The latest data show that since 2010, the Adaptation Fund has directed $532 million to 80

concrete adaptation projects in the most vulnerable communities of developing countries,

serving 5.8 million direct beneficiaries.

What is Adaptation fund?

• Established under the Kyoto Protocol of the UN Framework Convention on

Climate Change.

• It finances projects and programmes that help vulnerable communities in

developing countries adapt to climate change.

• Initiatives are based on country needs, views and priorities.

Financing:

• The Fund is financed in part by government and private donors, and also from a

two percent share of proceeds of Certified Emission Reductions (CERs) issued

under the Protocol’s Clean Development Mechanism projects.

Governance:

• The Fund is supervised and managed by the Adaptation Fund Board (AFB). The

AFB is composed of 16 members and 16 alternates and meets at least twice a year.

• The World Bank serves as trustee of the Adaptation Fund on an interim basis.

https://www.insightsonindia.com/2019/12/07/adaptation-fund/

83. Who among the following is known as Father of Indian archaeology?

(a) James Princep

(b) Max Mueller

(c) Sir William Jones

(d) Alexander Cunningham

Solution: D

The plight of Harappa

• Although Harappa was the first site to be discovered, it was badly destroyed by

brick robbers. As early as 1875, Alexander Cunningham, the first Director-General

of the Archaeological Survey of India (ASI), often called the father of Indian

archaeology, noted that the amount of brick taken from the ancient site was enough

to lay bricks for “about 100 miles” of the railway line between Lahore and Multan.

Thus, many of the ancient structures at the site were damaged. In contrast,

Mohenjodaro was far better preserved.

Page 82: SIMPLYFYING IAS EXAM PREPARATION - INSIGHTSIAS · 2020-04-13 · Insta 75 Days Revision Plan for UPSC Civil Services Prelims – 2020 This document is the compilation of 100 questions

INSTA 75 Days REVISION PLAN for Prelims 2020 - InstaTests

www.insightsonindia.com 80 Insights IAS

84. Consider the following statements regarding inscriptions

1. Asoka was the first ruler who inscribed his messages to his subjects and

officials on stone surfaces.

2. He used the inscriptions to proclaim what he understood to be Dhamma.

Which of the statements given above is/are correct?

(a) 1 only

(b) 2 only

(c) Both 1 and 2

(d) Neither 1 nor 2

Solution: C

• Asoka was the first ruler who inscribed his messages to his subjects and officials on

stone surfaces – natural rocks as well as polished pillars. He used the inscriptions

to proclaim what he understood to be dhamma. This included respect towards

elders, generosity towards Brahmanas and those who renounced worldly life,

treating slaves and servants kindly, and respect for religions and traditions other

than one’s own.

85. Consider the following pairs

Sea Shares border with

1. Aral Sea : Uzbekistan and Kazakhstan

2. Red Sea : Saudi Arabia and Oman

3. Black Sea : Turkey and Russia

Which of the pairs given above is/are correctly matched?

(a) 1 and 2 only

(b) 2 only

(c) 3 only

(d) 1 and 3 only

Solution: D

Page 83: SIMPLYFYING IAS EXAM PREPARATION - INSIGHTSIAS · 2020-04-13 · Insta 75 Days Revision Plan for UPSC Civil Services Prelims – 2020 This document is the compilation of 100 questions

INSTA 75 Days REVISION PLAN for Prelims 2020 - InstaTests

www.insightsonindia.com 81 Insights IAS

Page 84: SIMPLYFYING IAS EXAM PREPARATION - INSIGHTSIAS · 2020-04-13 · Insta 75 Days Revision Plan for UPSC Civil Services Prelims – 2020 This document is the compilation of 100 questions

INSTA 75 Days REVISION PLAN for Prelims 2020 - InstaTests

www.insightsonindia.com 82 Insights IAS

86. Consider the following statements regarding Prayaga Prashasti

1. It was composed in Sanskrit by Harishena, the court poet of Samudragupta.

2. It describes the circumstances of Samudragupta’s accession, his military

campaigns in north India and the Deccan.

Which of the statements given above is/are correct?

(a) 1 only

(b) 2 only

(c) Both 1 and 2

(d) Neither 1 nor 2

Solution: C

The Prayaga Prashasti (also known as the Allahabad Pillar Inscription) composed in

Sanskrit by Harishena, the court poet of Samudragupta, arguably the most powerful of the

Gupta rulers (c. fourth century CE), is a case in point.

The most important source for the reign of Samudragupta is the Allahabad Pillar

inscription. It describes his personality and achievements. This inscription is engraved on

an Asokan pillar. It is written in classical Sanskrit, using the Nagari script. It consists of

33 lines composed by Harisena. It describes the circumstances of Samudragupta’s

Page 85: SIMPLYFYING IAS EXAM PREPARATION - INSIGHTSIAS · 2020-04-13 · Insta 75 Days Revision Plan for UPSC Civil Services Prelims – 2020 This document is the compilation of 100 questions

INSTA 75 Days REVISION PLAN for Prelims 2020 - InstaTests

www.insightsonindia.com 83 Insights IAS

accession, his military campaigns in north India and the Deccan, his relationship with

other contemporary rulers, and his accomplishments as a poet and scholar.

87. Consider the following statements regarding Coins

1. The first gold coins were issued by the Guptas.

2. The first coins to bear the names and images of rulers were issued by the

Indo-Greeks.

Which of the statements given above is/are correct?

(a) 1 only

(b) 2 only

(c) Both 1 and 2

(d) Neither 1 nor 2

Solution: B

Coins and kings

• To some extent, exchanges were facilitated by the introduction of coinage. Punch-

marked coins made of silver and copper (c. sixth century BCE onwards) were

amongst the earliest to be minted and used.

• These have been recovered from excavations at a number of sites throughout the

subcontinent. Numismatists have studied these and other coins to reconstruct

possible commercial networks. Attempts made to identify the symbols on

punchmarked coins with specific ruling dynasties, including the Mauryas, suggest

that these were issued by kings. It is also likely that merchants, bankers and

townspeople issued some of these coins.

• The first coins to bear the names and images of rulers were issued by the Indo-

Greeks, who established control over the north-western part of the subcontinent c.

second century BCE.

• The first gold coins were issued c. first century CE by the Kushanas. These were

virtually identical in weight with those issued by contemporary Roman emperors

and the Parthian rulers of Iran, and have been found from several sites in north

India and Central Asia.

88. Consider the following statements

1. Indus Water Treaty was brokered by the World Bank

2. As per the treaty, control over three eastern rivers Ravi, Beas and Sutlej was

given to Pakistan.

Which of the statements given above is/are correct?

Page 86: SIMPLYFYING IAS EXAM PREPARATION - INSIGHTSIAS · 2020-04-13 · Insta 75 Days Revision Plan for UPSC Civil Services Prelims – 2020 This document is the compilation of 100 questions

INSTA 75 Days REVISION PLAN for Prelims 2020 - InstaTests

www.insightsonindia.com 84 Insights IAS

(a) 1 only

(b) 2 only

(c) Both 1 and 2

(d) Neither 1 nor 2

Solution: A

The Indus Waters Treaty is a water-distribution treaty between India and Pakistan,

brokered by the World Bank to use the water available in the Indus System of Rivers

located in India.

• The Indus Waters Treaty (IWT) was signed in Karachi on September 19, 1960 by

the first Prime Minister of India Pandit Jawaharlal Nehru and then President of

Pakistan Ayub Khan.

• According to this agreement, control over the water flowing in three “eastern

rivers” of India — the Beas, the Ravi and the Sutlej with the mean annual flow of

33 million acre-feet (MAF) — was given to India, while control over the water

flowing in three “western rivers” of India — the Indus, the Chenab and the Jhelum

with the mean annual flow of 80 MAF — was given to Pakistan.

89. Consider the following statements regarding Middle Stone Age

1. Stone tools often more than five centimeters in size, called microliths were

found.

2. There seems to have been a shift from small animal hunting to big animal

hunting.

Which of the statements given above is/are correct?

(a) 1 only

(b) 2 only

(c) Both 1 and 2

(d) Neither 1 nor 2

Solution: D

Mesolithic or Middle Stone Age

• The next stage of human life is called Mesolithic or Middle Stone Age which falls

roughly from 10000 B.C. to 6000 B.C. It was the transitional phase between the

Page 87: SIMPLYFYING IAS EXAM PREPARATION - INSIGHTSIAS · 2020-04-13 · Insta 75 Days Revision Plan for UPSC Civil Services Prelims – 2020 This document is the compilation of 100 questions

INSTA 75 Days REVISION PLAN for Prelims 2020 - InstaTests

www.insightsonindia.com 85 Insights IAS

Paleolithic Age and Neolithic Age. Mesolithic remains are found in Langhanj in

Gujarat, Adamgarh in Madhya Pradesh and also in some places of Rajasthan,

Utter Pradesh and Bihar. The paintings and engravings found at the rock shelters

give an idea about the social life and economic activities of Mesolithic people. In

the sites of Mesolithic Age, a different type of stone tools is found. These are tiny

stone artifacts, often not more than five centimeters in size, and therefore called

microliths. The hunting-gathering pattern of life continued during this period.

• However, there seems to have been a shift from big animal hunting to small animal

hunting and fishing. The use of bow and arrow also began during this period. Also,

there began a tendency to settle for longer periods in an area. Therefore,

domestication of animals, horticulture and primitive cultivation started. Animal

bones are found in these sites and these include dog, deer, boar and ostrich.

Occasionally, burials of the dead along with some microliths and shells seem to

have been practiced.

90. Consider the following statements regarding economic conditions during Rig Vedic

periods

1. The Rig Vedic Aryans were pastoral people.

2. Their wealth was estimated in terms of their cattle.

3. Workers in metal made a variety of articles with copper, bronze and iron.

Which of the statements given above is/are correct?

(a) 1 and 2 only

(b) 2 and 3 only

(c) 1 and 3 only

(d) 1, 2 and 3

Solution: D

Economic Condition

• The Rig Vedic Aryans were pastoral people and their main occupation was cattle

rearing. Their wealth was estimated in terms of their cattle. When they

permanently settled in North India they began to practice agriculture. With the

knowledge and use of iron they were able to clean forests and bring more lands

under cultivation. Carpentry was another important profession and the availability

of wood from the forests cleared made the profession profitable. Carpenters

produced chariots and ploughs. Workers in metal made a variety of articles with

copper, bronze and iron.

• Spinning was another important occupation and cotton and woolen fabrics were

made. Goldsmiths were active in making ornaments. The potters made various

kinds of vessels for domestic use. Trade was another important economic activity

Page 88: SIMPLYFYING IAS EXAM PREPARATION - INSIGHTSIAS · 2020-04-13 · Insta 75 Days Revision Plan for UPSC Civil Services Prelims – 2020 This document is the compilation of 100 questions

INSTA 75 Days REVISION PLAN for Prelims 2020 - InstaTests

www.insightsonindia.com 86 Insights IAS

and rivers served as important means of transport. Trade was conducted on barter

system. In the later times, gold coins called nishka were used as media of exchange

in large transactions.

91. Xaxa committee, sometime seen in the news, is related to

(a) Scheduled Caste

(b) Scheduled Tribes

(c) Minorities

(d) None of the above

Solution: B

The Government has set up a high level Committee (Xaxa Committee) to look into the

current socio-economic, health and educational status of tribals in the country.

• The objective of Committee was to prepare a position paper on the present socio-

economic, health and educational status of Scheduled Tribes and suggest a way

forward.

• The Committee’s objective was also to suggest policy initiative as well as effective

outcome-oriented measures to improve development indicators and strengthen

public service delivery to STs and other tribal population.

https://pib.gov.in/newsite/PrintRelease.aspx?relid=112080

92. Consider the following statements regarding Bimbisara

1. Bimbisara was a contemporary of Vardhamana Mahavira and Gautama

Buddha.

2. Bimbisara belonged to the Haryanka dynasty.

Which of the statements given above is/are correct?

(a) 1 only

(b) 2 only

(c) Both 1 and 2

(d) Neither 1 nor 2

Solution: C

Page 89: SIMPLYFYING IAS EXAM PREPARATION - INSIGHTSIAS · 2020-04-13 · Insta 75 Days Revision Plan for UPSC Civil Services Prelims – 2020 This document is the compilation of 100 questions

INSTA 75 Days REVISION PLAN for Prelims 2020 - InstaTests

www.insightsonindia.com 87 Insights IAS

Bimbisara (546 – 494 B.C.)

• Bimbisara belonged to the Haryanka dynasty. He consolidated his position by

matrimonial alliances. His first matrimonial alliance was with the ruling family of

Kosala. He married Kosaladevi, sister of Prasenajit. He was given the Kasi region

as dowry which yielded large revenue. Bimbisara married Chellana, a princess of

the Licchavi family of Vaisali. This matrimonial alliance secured for him the safety

of the northern frontier. Moreover, it facilitated the expansion of Magadha

northwards to the borders of Nepal. He also married Khema of the royal house of

Madra in central Punjab. Bimbisara also undertook many expeditions and added

more territories to his empire. He defeated Brahmadatta of Anga and annexed that

kingdom. He maintained friendly relations with Avanti. He had also efficiently

reorganized the administration of his kingdom.

• Bimbisara was a contemporary of both Vardhamana Mahavira and Gautama

Buddha. However, both religions claim him as their supporter and devotee. He

seems to have made numerous gifts to the Buddhist Sangha.

93. Consider the following statements regarding book Gathasaptasati

1. It was written by Simuka, founder of the Satavahana dynasty.

2. It contains around 700 verses in Prakrit language.

Which of the statements given above is/are correct?

(a) 1 only

(b) 2 only

(c) Both 1 and 2

(d) Neither 1 nor 2

Solution: B

The founder of the Satavahana dynasty was Simuka. He was succeeded by Krishna, who

extended the kingdom up to Nasik in the west. The third king was Sri Satakarni. He

conquered western Malwa and Berar. He also performed asvamedha sacrifices. The

seventeenth king of the Satavahana dynasty was Hala. He reigned for a period of five

years. Hala became famous for his book Gathasaptasati, also called Sattasai. It contains

700 verses in Prakrit language.

94. Consider the following statements regarding Most Favoured Nation Status

1. It refers to the phenomenon where any country grants one country a special

favour such as a lower customs duty rate for one of their products the same

would need to be extended to all other WTO members.

2. It applies only to trading of goods between the countries.

Page 90: SIMPLYFYING IAS EXAM PREPARATION - INSIGHTSIAS · 2020-04-13 · Insta 75 Days Revision Plan for UPSC Civil Services Prelims – 2020 This document is the compilation of 100 questions

INSTA 75 Days REVISION PLAN for Prelims 2020 - InstaTests

www.insightsonindia.com 88 Insights IAS

Which of the statements given above is/are correct?

(a) 1 only

(b) 2 only

(c) Both 1 and 2

(d) Neither 1 no 2

Solution: A

• Under the World Trade Organisation (WTO) agreements, countries cannot

normally discriminate between their trading partners.

• If any country grants one country a special favour such as a lower customs duty

rate for one of their products the same would need to be extended to all other WTO

members. This principle is known as most-favoured-nation (MFN) treatment.

• MFN is so important a principle that it is the first article of the General Agreement

on Tariffs and Trade (GATT), which governs trade in goods. MFN is also a

priority in the General Agreement on Trade in Services (GATS) (Article 2) and

the Agreement on Trade-Related Aspects of Intellectual Property Rights

(TRIPS) (Article 4). Together, those three agreements cover all three main areas of

trade handled by the WTO.

http://www.arthapedia.in/index.php?title=Most-favoured-nation_(MFN)

95. During the period of Guptas, the designation ‘Sandivigraha’ was used for

(a) Minister for foreign affairs

(b) Chief minister

(c) Commander-in-chief

(d) Minister for taxes

Solution: A

According inscriptions, the Gupta kings assumed titles like Paramabhattaraka,

Maharajadhiraja, Parameswara, Samrat and Chakravartin. The king was assisted in his

administration by a council consisting of a chief minister, a Senapati or commander-in-

chief of the army and other important officials. A high official called Sandivigraha was

mentioned in the Gupta inscriptions, most probably minister for foreign affairs.

Page 91: SIMPLYFYING IAS EXAM PREPARATION - INSIGHTSIAS · 2020-04-13 · Insta 75 Days Revision Plan for UPSC Civil Services Prelims – 2020 This document is the compilation of 100 questions

INSTA 75 Days REVISION PLAN for Prelims 2020 - InstaTests

www.insightsonindia.com 89 Insights IAS

96. Consider the following statements regarding Human Space Flight Programme

(HSP)

1. It is a proposal by ISRO to develop and launch the ISRO Orbital Vehicle,

which would carry crew members to geosynchronous orbit.

2. The spaceship would be launched by the GSLV Mk-III launcher.

Which of the statements given above is/are correct?

(a) 1 only

(b) 2 only

(c) Both 1 and 2

(d) Neither 1 nor 2

Solution: B

The Indian Human Spaceflight Programme (HSP) is a proposal by ISRO to develop and

launch the ISRO Orbital Vehicle, which would carry a two-member crew to the Low

Earth Orbit.

• The programme envisages the development of a fully autonomous orbital vehicle

carrying two crew members to about 300 km low earth orbit and their safe return.

• The spaceship would be launched by the GSLV MkIII launcher. Pre-project

activities have been initiated to study and develop critical technologies for this

mission

https://www.vssc.gov.in/VSSC/index.php/advanced-technologies/human-space-flight

97. Consider the following statements regarding book Ashtangasamgraha

1. Susruta was the author of Ashtangasamgraha.

2. It contains summary of the eight branches of medicine.

Which of the statements given above is/are correct?

(a) 1 only

(b) 2 only

(c) Both 1 and 2

(d) Neither 1 nor 2

Solution: B

Page 92: SIMPLYFYING IAS EXAM PREPARATION - INSIGHTSIAS · 2020-04-13 · Insta 75 Days Revision Plan for UPSC Civil Services Prelims – 2020 This document is the compilation of 100 questions

INSTA 75 Days REVISION PLAN for Prelims 2020 - InstaTests

www.insightsonindia.com 90 Insights IAS

In the field of medicine, Vagbhata lived during this period. He was the last of the great

medical trio of ancient India. The other two scholars Charaka and Susruta lived before

the Gupta age. Vagbhata was the author Ashtangasamgraha (Summary of the eight

branches of medicine).

98. Which of the following organizations has launched World Vision Report?

(a) World Health Organization

(b) Doctors without borders

(c) World Vision International

(d) International Agency for the Prevention of Blindness (IAPB)

Solution: A

The first-ever World Vision Report was recently released by WHO.

Highlights of Report:

• More than a quarter of the world’s population — some 2.2 billion people — suffer

from vision impairment.

• Presbyopia, a condition in which it is difficult to see nearby objects, affects 1.8

billion people. This condition occurs with advancing age.

• Trachoma is caused due to bacterial infection in the eye. Many countries have

eliminated it, including India.

• The prevalence of vision impairment in low- and middle-income regions was

estimated by the report to be four times higher than in high-income regions

• Myopia is the highest in high-income countries of the Asia-Pacific region (53.4 per

cent), closely followed by East Asia (51.6 per cent).

99. Consider the following statements regarding Megalithic Burials

1. The Iron Age of the southern peninsula is often related to Megalithic

Burials.

2. Black and red pottery, iron artifacts such as hoes and sickles and small

weapons were found in the burial pits.

Which of the statements given above is/are correct?

(a) 1 only

(b) 2 only

(c) Both 1 and 2

(d) Neither 1 nor 2

Page 93: SIMPLYFYING IAS EXAM PREPARATION - INSIGHTSIAS · 2020-04-13 · Insta 75 Days Revision Plan for UPSC Civil Services Prelims – 2020 This document is the compilation of 100 questions

INSTA 75 Days REVISION PLAN for Prelims 2020 - InstaTests

www.insightsonindia.com 91 Insights IAS

Solution: C

The Iron Age of the southern peninsula is often related to Megalithic Burials. Megalith

means Large Stone. The burial pits were covered with these stones. Such graves are

extensively found in South India. Some of the important megalithic sites are Hallur and

Maski in Karnataka, Nagarjunakonda in Andhra Pradesh and Adichchanallur in Tamil

Nadu. Black and red pottery, iron artifacts such as hoes and sickles and small weapons

were found in the burial pits.

100. Consider the following the statements regarding Rangdum monastery:

1. It is located in the state of Sikkim

2. It is a Tibetan Buddhist monastery belonging to the Gelugpa sect

Which of the statements given above is/are correct?

(a) 1 only

(b) 2 only

(c) Both 1 and 2

(d) Neither 1 nor 2

Solution: B

Rangdum Monastery is a Tibetan Buddhist monastery belonging to the Gelugpa sect,

situated on top of a small but steep sugarloaf hill at an altitude of 4,031 m at the head of

the Suru Valley, in Ladakh. It is next to the tiny village of Julidok, and about 25 km from

the 4,400 m Pensi La, which leads into Zanskar

Extra Learning:

The Gelugpa (Gelug) Sect was founded by Tsongkhapa, a reformist monk and a great

Tibetan scholar, and headed by the Dalai Lama. Tsongkhapa (1357-1419) began studying

with a local Sakya lama at a very young age. He travelled to central Tibet and studied in

all the major monasteries acquiring knowledge about philosophy, Tibetan medicine,

Mahamurda practices, and Atisha’s tantric yoga.

Tsongkhapa was particularly inspired by Madhyamika teachings of Nagarjuna. He

preached monastic discipline and encouraged return to the conservative doctrines of

Buddhism. Regarded as the manifestation of Bodhisattva of Wisdom, Jampelyan

Page 94: SIMPLYFYING IAS EXAM PREPARATION - INSIGHTSIAS · 2020-04-13 · Insta 75 Days Revision Plan for UPSC Civil Services Prelims – 2020 This document is the compilation of 100 questions

INSTA 75 Days REVISION PLAN for Prelims 2020 - InstaTests

www.insightsonindia.com 92 Insights IAS

(Manjushri), Tsongkhapa’s followers built him the Ganden Monastery in Netang near

Lhasa in 1409. After his death, Tsongkhapa’s students built a new school of Tibetan

Buddhism based on his teachings and named it ‘Gelug’ which means virtuous tradition.

Page 95: SIMPLYFYING IAS EXAM PREPARATION - INSIGHTSIAS · 2020-04-13 · Insta 75 Days Revision Plan for UPSC Civil Services Prelims – 2020 This document is the compilation of 100 questions

www.insightsonindia.com INSIGHTS IAS

TESTIMONIALS INSTA 75 Days REVISION PLAN for Prelims 2020 – InstaTests

http://disq.us/p/282plo8

http://disq.us/p/282palg

http://disq.us/p/2839b2u

http://disq.us/p/28595xc

http://disq.us/p/2857k0t

http://disq.us/p/2857hhq

http://disq.us/p/2856dfp

http://disq.us/p/2858me1

http://disq.us/p/28592b1

Page 96: SIMPLYFYING IAS EXAM PREPARATION - INSIGHTSIAS · 2020-04-13 · Insta 75 Days Revision Plan for UPSC Civil Services Prelims – 2020 This document is the compilation of 100 questions

www.insightsonindia.com INSIGHTS IAS

http://disq.us/p/2855z70

http://disq.us/p/2855vwp

http://disq.us/p/2855yka

http://disq.us/p/2852u5r

http://disq.us/p/2854jny

http://disq.us/p/285733x

http://disq.us/p/2856vij